You are on page 1of 71

1. Most common nerve injured in supracondylar fracture humerus?

 
a. Median 
b. Radial 
c. Ulnar 
d. Anterior interosseus nerve 
Ans: D 
2. Earliest symptom of GERD in an infant is? 
A. Respiratory distress answer 
B. Upper GI bleed 
C. regurgitation 
D.obstruction 

Ans: C REGURGITATION.... 
3. Basal metabolic rate is closely associated with? 
A. Lean body mass 
B. Body surface area 
C. Body mass index 
D. Body weight 
Ans: A Lean Body mass (REPEAT...shall not spend much time on repeats) 
4. 4. A girl presented with recurrent occipital headache associated with ataxia and vertigo. Mother also has similar
complaints. Most probable diagnosis is? 
A. Vestibular neuronitis 
B. Basillar migraine 
C. TIA 
D. – 
Ans: b Basilar migraine(REPEAT)
5. Drug of choice for central Diabetes Insipidus is? 
a. Desmopressin 
b. Leuperolide 
c. Thiazide diuretics 
d. – 
Ans: A Desmopressin 
Reference: Harrispn 17th ed 
The signs and symptoms of uncomplicated pituitary DI can be eliminated completely by treatment with desmopressin
(DDAVP), a synthetic analogue of AVP (Fig. 334-1). It acts selectively at V2 receptors to increase urine concentration and
decrease urine flow in a dose-dependent manner (Fig. 334-4). It is also more resistant to degradation than AVP and has
a three- to fourfold longer duration of action. Desmopressin (DDAVP) can be given by IV or SC injection, nasal inhalation,
or oral tablet. 
6. A 32 year old mountaineer has a hematocrit of 70%. What is the possible explanation? 
A. Polycythemia with relative dehydration 
B. High altitude cerebral oedema 
C. High altitude pulmonary oedema 
D. Hemodilution 

Ans.A Polycythemia and relative dehydration 

Reference:Harrison 17th ed 


Polycythemia can be spurious (related to a decrease in plasma volume; Gaisbock's syndrome), primary, or secondary in
origin. The secondary causes are all associated with increases in EPO levels: either a physiologically adapted appropriate
elevation based on tissue hypoxia (lung disease, high altitude, CO poisoning, high-affinity hemoglobinopathy) or an
abnormal overproduction (renal cysts, renal artery stenosis, tumors with ectopic EPO production). A rare familial form of
polycythemia is associated with normal EPO levels but hyperresponsive EPO receptors due to mutations 

7. Tetracycline is used in the prophylaxis of which of the following diseases? 


a. Cholera 
b. Brucellosis 
c. Leptospirosis 
d. Meningitis 
Ans: a Cholera

 8. A 7 month old child has bouts of cough ending with a whoop. What is the best way to confirm the diagnosis? 
A. Nasophayngeal swab 
B. Cough plate culture 
C. Tracheal aspirate 
D. – 

Ans: a Nasopharyngeal swab 


Harrison 17th ed 

The diagnosis is Pertusis and we have to find the best specimen to confirm the diagnosis. 

Culture of nasopharyngeal secretions remains the gold standard of diagnosis, although DNA detection by polymerase
chain reaction (PCR) is replacing culture in many laboratories because of increased sensitivity and quicker results. The
best specimen is collected by nasopharyngeal aspiration, in which a fine flexible plastic catheter attached to a 10-mL
syringe is passed into the nasopharynx and withdrawn while gentle suction is applied. 

9. Aflatoxin is produced by? 


A. Aspergillus flavus 
B. Aspergillus niger 
C. Candida 
D. – 
Ans: a Aspergillus Flavus 

MYCOTOXINS 
Many fungi produce poisonous substances called mycotoxins that can cause acute or chronic intoxication and damage.
The mycotoxins are secondary metabolites, and their effects are not dependent on fungal infection or viability. A variety
of mycotoxins are produced by mushrooms (eg, amanita species), and their ingestion results in a dose-related disease
called mycetismus. Cooking has little effect on the potency of these toxins, which may cause severe or fatal damage to
the liver and kidney. Other fungi produce mutagenic and carcinogenic compounds that can be extremely toxic for
experimental animals. One of the most potent is aflatoxin, which is elaborated by Aspergillus flavus and related molds
and is a frequent contaminant of peanuts, corn, grains, and other foods. 
10). Most important prognostic factor in congenital diaphragmatic hernia? 
A. Pulmonary hypertension 
B. Timing of surgery 
C. Size of defect 
D. – 
Ans: A Pulmonary hypertension 
Reference: Nelson 18th ed 
Overall survival of live-born infants is 67%. The incidence of spontaneous fetal demise with CDH diagnosis is 7–10%.
Relative predictors of a poor prognosis include an associated major anomaly, symptoms before 24 hr of age, severe
pulmonary hypoplasia, herniation to the contralateral lung, and the need for ECMO. 
Serious sequelae include pulmonary function changes, neurodevelopmental delays, and growth retardation. Pulmonary
problems continue to be a source of morbidity for long-term survivors of CDH. Children receiving CDH repair studied at
6–11 yr of age demonstrate significant decreases in forced expiratory flow at 50% of vital capacity and decreased peak
expiratory flow. Both obstructive and restrictive patterns can occur. Those without severe pulmonary hypertension and
barotrauma do the best. Those at highest risk include children who required ECMO and patch repair, but the data clearly
show that non-ECMO CDH survivors also require frequent attention to pulmonary issues. At discharge, up to 20% of
infants require oxygen, but only 1–2% require it past 1 yr of age.

11. A surgeon removed the part of liver to the left of the falciform ligament. Which segments have been removed? 
A. 1 & 4a 
B. 2 & 3 
C. 1 & 4b 
D. 5 & 6 
Ans: B 2 & 3(REPEAT) 
12. Punnett square is used for 
a. Finding genotype of offspring 
b. Statistical analysis 
c. – 
d. – 
Ans:a Finding the genotype of the offspring 
Reference: The Tennessee Gateway Science 
Punnett square is a Diagram used to identify possible combinations of recessive and dominant alleles in OFFSPRING.To create a
punnette square divide a suare into 4 parts and write the letters that represent the alleles of one parent on top of the suare and the
alleles of the other parent on the side of the square..Just like we make a 2X2 table for statistics questions.Combine the allele of one
parent with the other and work out the 4 possible combinations.The punnett square does not give the exact information about the
offspring but instead gives the probability. 
13. Cavitation is seen in?  
A. Mycolplasma pneumonia  
B. Tuberculous pneumonia  
C. Streptococcal pneumonia  
D.Staphylococcus pneumonia 

Ans: D Staphylococcal Pneumonia(REPEAT) 

14. In PSUDOHYPOPARATHYROIDISM true is?(NEED FINETUNING OF STEM) 


a. Decreased cAMP 
b. Decreased IP3  
c. Increased gtpas activity/Gain of function mutation in GTPase 
d.--- 

Ans:a Decreased cAMP 

Reference: Harrisson 17th ed 


Individuals with Pseudohypoparathyroidism 1, the most common of the disorders, show a deficient urinary cyclic AMP response to
administration of exogenous PTH. Patients with PHP-I are divided into type a, with AHO and reduced amounts of Gs in in vitro
assays with erythrocytes, and type b, lacking AHO and with normal amounts of Gs in erythrocytes. There is a third type (PHP-Ic,
reported in a few patients) that differs from PHP-Ia only in having normal erythrocyte levels of Gs despite having AHO,
hypocalcemia, and decreased urinary cyclic AMP responses to PTH (presumably with a post-Gs defect in adenyl cyclase
stimulation). 
15. Difference between follicular carcinoma and follicular adenoma is?  
A. Vascular invasion  
B. Mitosis 
C. Nuclear pleomorphism 
D. Tubule formation 
Ans: a Vacular invasion (REPEAT) 
 16. Antidepressant drug that can be used in nocturnal eneuresis?  
A. Imipramine  
B. Fluoxamine  
C.  
D.  
Ans: A Imipramine 

17. A female patient presented with depressed mood, loss of appetite and no interest in surroundings. There is associated
insomnia. The onset of depression was preceeded by a history of business loss and immediately soon after it she developed the
followiung symptoms for the past 1yr . True is? 
A. No treatment is necessary as it is due to business loss 
B. SSRI is the most efficacious of the available drugs 
C. Start SSRI treatment based on side effect profile 
D. Combination therapy of 2 anti depressant drugs 

Ans:C Treatment is started based on the side effect profile 

18. An Infant is brought to casualty with reports of violent shaking by parents. Most characteristic injury is? 
A. Long bone fracture answer 
B. Ruptured spleen 
C. Subdural hematoma 
D. Skull bone fracture 

Ans: Subdural hematoma 

Reference: Forensic Medicine By Reddy 

Under the Topic Battered Baby syndrome Reddy talks about SUBDURAL HEMATOMA being the MOST CHARACTERISTIC FEATURE of
violent shaking of an infant by the Parent...Its called INFANTILE WHIPLASH SYNDROME....In Battered baby syndrome multiple Long
bone fractures at various stages of healing may be seen and not in infantile whiplash syndrome.. 
19. Gun powder on clothing can be visualized by? 
A. Magnifying lens 
B. UV rays 
C. Infrared rays 
D. Dye 
Ans: B UV Rays (REPEAT) 
20. Capsular antibody protection is seen in all except? 
A. Neisseria meningitidis 
B. Pneumococcus 
C. Bordetella pertussis 
D. Haemophilus influenza 

Ans: C Bordetella pertusis 

21. Signature fracture refers to? 


A. Depressed skull fracture 
B. Suture displacement fracture 
C. Contrecoup injury 
D. Fracture at foramen magnum 
Ans: Depressed fracture (REPEAT) I don’t think the question needs an explanation 
22. Which among the following is most frequently seen in anti phospholipid antibody syndrome?(Pls CORRECT Q) 
A. Beta 2 microglobulin antibody 
B. Anti nuclear antibody 
C. Anti centromere antibody 
D. Anti beta 2 glycoprotein antibody 
Ans: D Anti beta 2 GP Ab 
The antiphospholipid antibody syndrome (APS) may be defined as the occurrence of arterial or venous thrombosis or recurrent
miscarriage in association with laboratory evidence of persistent antiphospholipid antibody. The antibody may manifest itself as
either a lupus anticoagulant detected by clotting tests or an anticardiolipin antibody (ACA) detected by immunoassay. The
syndrome may be associated with an autoimmune disorder, especially SLE (secondary), or may occur independently of other
autoimmune disorders (primary). A large proportion of antiphospholipid antibodies are actually directed against complexes of
phospholipid with protein, most notably beta-2-glycoprotein I. The ACA immunoassay detects both these clinically significant
antibodies and antibodies directed purely against cardiolipin and not beta-2-glycoprotein I. The latter antibodies are transient,
and are often associated with infection but not thrombosis. A positive anti-b2 GPI assay is evidence in favor of APS, once
persistence is demonstrated by repeating the assay after eight weeks. 
23. What factor is responsible for deciding whether an antibody will remain membrane bound or get secreted? 
A. RNA splicing 
B. Class switching 
C. Differential RNA processing 
D. Allelic exclusion 
Ans: C Differential RNA Processing(REPEAT) 
24. Blood Chimerism is associated with? 
A. Monochorionic monoamniotic twins 
B. Monochorionic diamniotic twins 
C. Singleton pregnancy 
D. Vanishing twin 
Ans: B Monochorionic diamniotic twins (no doubt if these were the options) 
Explanation: Reference: 
“ Journal Ref: Blood Chimerism in a Dizygotic Dichorionic Pregnancy” 

“Blood chimerism in monochorionic twins conceived by induced ovulation: Case report” 


Journal says “ Blood chimerism “is more common in monochorionic dizygotic(MCDZ)twins and rare in dichorionic twins 
“Cases in which a monochorionic placenta occurs in a twin pregnancy,vascular anstomosis is well described and can lead to blood
chimerism as well as twin twin transfusion syndrome.In contrast interplacental vascular communications occur only with very rare
exceptions in a dichorionic placenta” 
25. In expectant management of placenta praevia, all are done except? 
A. Cervical encirclage 
B. Anti D 
C. Corticosteroids 
D. Blood transfusion 
Ans: a Cervical encirclage 
Reference: Williams Obstetrics 
The treatment of classical cervical incompetence is cerclage. The operation is performed to surgically reinforce 
the weak cervix by some type of purse-string suturing. Bleeding, uterine contractions, or ruptured membranes are usually
contraindications for cerclage.
Post Options: Add to favourite . Tell a friend . Email me when a reply is posted

26. Which drug is not used during delivery in a woman with rheumatic heart disease ? 
A. Methylergometrine 
B. Carboprost 
C. Syntocin 
D. Misoprostol 
Ans: A Methylergometrine(REPEAT) 
27. Which is not an autoimmune disease? 
A. SLE 
B. Grave's disease 
C. Myasthenia gravis 
D. Sickle cell disease 
Ans: D Sickle Cell Disease(I don’t think an explanation is needed..Sickle cell ds is a hemoglobinopathy and not an autoimmune ds) 
28. All are true regarding selective estrogen receptor downregulator (SERD),fulvestrant except? 
A. Used for breast cancer 
B. Is a selective oestrogen antagonist 
C. Is slower acting, safer, LESS effective than SERM 
D. Given as once a month im dose 
Ans: C Its slower acting and less efficacious than SERM 
Reference:This Question has been taken line to line from Goodman Gilman Pharmacology as u will see below: 
Fulvestrant 
Fulvestrant (FASLODEX) is the first FDA approved agent in the new class of estrogen-receptor downregulators, which were
hypothesized to have an improved safety profile, faster onset, and longer duration of action than the SERMs due to their pure ER
antagonist activity (Robertson, 2002). Fulvestrant was approved in 2002 for postmenopausal women with hormone receptor-
positive metastatic breast cancer that has progressed despite antiestrogen therapy. 
Mechanism of Action 
Fulvestrant is a steroidal antiestrogen that binds to the ER with an affinity more than 100 times that of tamoxifen, inhibits its
dimerization, and increases its degradation. 
Preclinical studies suggest that as a consequence of this ER "downregulation," ER-mediated transcription is abolished, completely
suppressing the expression of estrogen-dependent genes (Howell et al., 2004b). This difference in the activity of fulvestrant likely
explains why fulvestrant demonstrates efficacy against tamoxifen-resistant breast cancer. 
However, the hypothesis that fulvestrant provides more effective antiestrogen activity than tamoxifen was not confirmed by a
clinical trial comparing fulvestrant (250 mg intramuscularly monthly) with tamoxifen (20 mg orally daily) as first-line therapy in
metastatic breast cancer (Howell et al., 2004a). 
Absorption, Fate, and Excretion 
Maximum plasma concentrations are reached about 7 days after intramuscular administration of fulvestrant and are maintained
over a period of 1 month. The plasma half-life is approximately 40 days. Steady-state concentrations are reached after 3 to 6
monthly injections. There is extensive and rapid distribution, predominantly to the extravascular compartment. 
Various pathways, similar to those of steroid metabolism including oxidation, aromatic hydroxylation, and conjugation, extensively
metabolize fulvestrant. CYP3A4 appears to be the only CYP isoenzyme involved in the oxidation of fulvestrant. Several preclinical
and clinical studies have confirmed that fulvestrant is not subject to CYP3A4 interactions that might affect the safety or efficacy of
the drug. The putative metabolites possess no estrogenic activity and only the 17-keto compound demonstrates a level of
antiestrogenic activity about 4.5 times less than that of fulvestrant. The major route of excretion is via the feces, with less than 1%
being excreted in the urine (Robertson and Harrison, 2004). 
Therapeutic Uses 
Fulvestrant typically is administered as a 250-mg intramuscular injection at monthly intervals. It is used in postmenopausal women
as antiestrogen therapy of hormone receptor-positive metastatic breast cancer after progression on first-line antiestrogen therapy
such as tamoxifen (Strasser-Weippl and Goss, 2004). Fulvestrant is at least as effective in this setting as the third-generation
aromatase inhibitor anastrozole. 
Fulvestrant 250 mg (administered as a once-monthly 5-ml intramuscular injection) also has been compared with tamoxifen 20 mg
(orally once daily) in a trial of postmenopausal women with ER-positive and/or progesterone receptor (PR)-positive or ER/PR-
unknown metastatic breast cancer who had not previously received endocrine or chemotherapy. There was no difference between
fulvestrant and tamoxifen in time to disease progression in either the entire study population or the subset of patients with ER-
and/or PR-positive disease. Observed differences in other efficacy endpoints favored tamoxifen, and fulvestrant equivalence was not
demonstrated (Vergote and Robertson, 2004). The long time to steady-state plasma levels for fulvestrant has brought into question
the results of existing studies, and trials are in progress to test the relative efficacy of giving an initial loading dose followed by
regular monthly injections. 
Clinical Toxicity 
Fulvestrant generally is well tolerated with the most common adverse events being nausea, asthenia, pain, vasodilation (hot
flushes), and headache. Injection site reactions, seen in about 7% of patients, are reduced by giving the injection slowly. In the
study comparing anastrozole and fulvestrant, quality-of-life outcome measures were maintained over time with no significant
difference between the drugs . 
29. A farmer developed a swelling in the inguinal region which later ulcerated. What stain can be used to detect bipolar stained
organisms? 
A. Albert's stain 
B. Waysons stain 
C. Ziehl neelsen stain 
D. Nigrosin stain 
Ans: B Wayson stain(REPEAT) 
30. An 8 year old boy completed 8 out of 10 day course of cefaclor. Now he developed a generalized erythmatic rash which is mildly
pruritic and lymphadenopathy. Diagnosis is? 
A. Kawasaki disease 
B. Type 3 hypersensitivity 
C. Anaphylaxis 
D. Infectious mononucleosis 
Ans: B Type 3 Hypersensitivity(REPEAT)
31. Rave drug is? 
A. Cannabis 
B. Cocaine 
C. Heroin 
D. Amphetamine 
Ans: D Amphetamine(MDMA) 
MDMA is also called Rave drug and ecstasy 
32. How to differentiate ASD from VSD in X-ray? 
A. Enlarged Left atrium 
B. Normal left atrium 
C. Pulmonary congestion 
D. Aortic shadow 
Ans: B Normal LA size 
LA remains normal in ASD despite volume overload since it can decompress through 2 outlets that is into RA and into LV.So LA
enlargement is not seen in ASD wheras it is seen in VSD 
33. Regarding an imbecile, all are true except? 
A. IQ is 50-60 B. Intellectual capacity equivalent to a child of 3-7 years of age 
C. Not able to take care of themselves 
D. Condition is congenital or acquired at an early age 
Ans: A IQ is 50-60 
Reference: Kaplan and saddock 

Q Range Classification 
70-80 Borderline deficiency 
50-69 Moron 
20-49 Imbecile 
below 20 Idiot 

34. Which is not true regarding diet modification recommended in high cardiovascular risk group? 
A. Cholesterol less then 100 mg/1000kcal/day 
B. Avoid alcohol 
C. Fat intake 10% of total calories 
D. Salt limitation to less than 5 gm 
Ans: C Fat intake <10% of total calories 
Reference: American Heart association 2006 guidelines 
These guidelines confirm the options 1/2/4...regarding option 3....it is the saturated fat content which should be less that 10%.the
total fat intake should be less than 30% of the overall calorie intake. 
IF THE OPTION 3 WAS....Saturated Fat intake less than 10% then the answer shall become Avoid alcohol...as AHA guidelines say
upto 2-3 drink per day is fine.... 
35. Maintenance dose of which of the following drugs is used worldwide for opioid dependence? 
A. Naltrexone 
B. Methadone 
C. lmipramine 
D. Disulfiram 
Ans: B Methadone 
Reference: Kaplan and saddock 
Methadone is a synthetic narcotic (an opioid) that substitutes for heroin and can be taken orally. When given to addicts to replace
their usual substance of abuse, the drug suppresses withdrawal symptoms. A daily dosage of 20 to 80 mg suffices to stabilize a
patient, although daily doses of up to 120 mg have been used. The duration of action for methadone exceeds 24 hours; thus, once-
daily dosing is adequate. Methadone maintenance is continued until the patient can be withdrawn from methadone, which itself
causes dependence. An abstinence syndrome occurs with methadone withdrawal, but patients are detoxified from methadone more
easily than from heroin. Clonidine (0.1 to 0.3 mg three to four times a day) is usually given during the detoxification period. 
Methadone maintenance has several advantages. First, it frees persons with opioid dependence from using injectable heroin and,
thus, reduces the chance of spreading HIV through contaminated needles. Second, methadone produces minimal euphoria and
rarely causes drowsiness or depression when taken for a long time. Third, methadone allows patients to engage in gainful
employment instead of criminal activity. The major disadvantage of methadone use is that patients remain dependent on a narcotic.

36. Best test/Gold standard test for assesing HCG function/action? 


A. Radioimmunoassay 
B. ELISA 
C. Latex test 
D. Bioassay 
Ans: D Bioassay(OPEN TO SCRUTINY......CHALLENGING REFERENCES INVITED) 
My explanation....Though Radioimmunoassay is more sensitive than Bioassay for quatifying an antigen ,Bioassay only can asses an
harmone both quantitatively and qualitatively....THE Question SPECIFICALLY asked about the BEST TEST OF FUNCTION which can
be assessed by BIOASSAY and not Radioimmunoassay.
37. Vitamin K is involved in the posttranswerlational modification of? 
A. Glutamate 
B. Aspartate 
C. -- 
D. – 
Ans: A Glutamate(REPEAT) 
38. Spinal anaesthesia is given at which level? 
A. L1-2 
B. L2-4 
C. S1 
D. Midline thoracic segments 
Ans: B L2-4 
Reference: Miller anesthesia(Procedureconsult) 
• Spinal anesthetics have their effects at the spinal cord, which originates at the foramen magnum of the skull and the brainstem
and extends caudally to the conus medullaris. The distal termination varies from about the level of the 3rd lumbar vertebrae (L3) in
infants to the lower border of L1 in adults. The spinal cord is surrounded by three membranes (from central to peripheral): the pia
mater, arachnoid mater, and dura mater. It is believed that the arachnoid mater is responsible for up to 90% of the resistance to
drug migration in and out of the CSF. Inside the subarachnoid space are the CSF, spinal nerves, a network of trabeculae between
the two membranes, and blood vessels supplying the spinal cord. Although the spinal cord ends at about L1 in adults, the
subarachnoid space continues to about the second sacral vertebrae (S2). 
• Posterior to the epidural space is the ligamentum flavum, which extends from the foramen magnum to the sacral hiatus.
Immediately posterior to the ligamentum flavum are the lamina and spinous processes of the vertebral bodies or the interspinous
ligaments. Posterior to these structures is the supraspinous ligament, which joins the vertebral spines. 
• Anatomic landmarks most important to performance of spinal anesthesia are the iliac crests, the midline of the back, and the
vertebral spinous processes. Palpation of the midline of the back identifies the spinous processes and vertebral interspaces in most
patients but may be difficult in obese patients. A line drawn between the upper borders of the iliac crests across the midline of the
back identifies the approximate level of L4 or the L4-L5 interspace. 
• Spinal anesthesia is usually performed at the level of the L3 or L4 vertebrae in the adult patient, because the spinal needle is
introduced below the level at which the spinal cord ends. 

39. a child presented with mild fever little breathlessness..... was treated and she improved over 4 days and later deteriorated
again with fever and more breathlessness. x ray showed hyperlucency. diagnosis? 
1.bronchiolitis obliterans 
2.alveolar proteinosois 
3.bronchitis 4. AsthmA 
Ans: a bronchiolitis obliterans(REPEAT) 
40. Which of the following passes through foramen magnum? 
A. Internal Carotid Artery 
B. Sympathetic chain 
C. Hypoglossal Nerve 
D. Vertebral Artery 
Ans: D Vertebral artery(REPEAT)

41. McKeon's theory on reduced prevalence of TB? 


A. Increased awareness and knowledge 
B. Medical advancement answer 
C. Behavioural modification 
D. Social and environmental factor 
Ans: D Social and environmental factor 
Reference:” McKeown and the Idea That Social Conditions Are Fundamental Causes of Disease Bruce G. Link, PhD and Jo C. Phelan,
PhD “ 
THE MCKEOWN THESIS STATES that the enormous increase in population and dramatic improvements in health that humans have
experienced over the past 2 centuries owe more to changes in broad economic and social conditions than to specific medical
advances or public health initiatives.1 The thesis gives center stage to social conditions as root causes of the health of populations.
On the basis of new data and numerous revisitations, however, Colgrove2 tells us that the thesis has been “overturned” and the
theory “discredited.” Whither, then, the idea that social conditions require prominence in any complete understanding of the health
of populations? When we turn away from “the thesis,” do we accept an “antithesis” asserting that the role of social conditions is
insignificant? 
42. Which among the following is not a cause of fasting hypoglycemia? 
A. Glucagon excess 
B. Glucose 6 phospatase deficiency 
C. Ureamia 
D. Glycogen synthase deficiency 
Ans: A Glucagon excess 
Explanation: I am not quoting any book...But i will explain...if anyone has a contradictory reference pls post... 
Glucose 6 phosphatase def leads to inability to mobilize glucose and hence can cause 
Glycogen synthase def leads to glycogen deficiency and hence during fasting patient goes hypo 
In uremia liver metabolism is hampered and hence hypo is possible 
In glucagon excess there is HYPERglycemia and hence the answer. 
43. Mineralocorticoid receptor is not present in? 
A. Liver 
B. Colon 
C. Hippocampus 
D. Kidney 
Ans:Liver(REPEAT) 
44. Prolonged treatment with INH leads to deficiency of? 
A. Pyridoxine 
B. Thiamine 
C. Pantothenic acid 
D. Niacin 
Ans: A Pyridoxine 
Reference: Harrison 17th ed table of vitamins 
Vitamin B6 
Defeciency symptoms: Seborrhea, glossitis convulsions, neuropathy, depression, confusion, microcytic anemia RDA:<0.2 mg
Factors contributing to deficiency:Alcoholism, isoniazid 

45. Which is the most reliable objective sign of identifying pulmonary plethora in chest X-ray? 
A. Diameter of the main pulmonay artery >16mm 
B. Diameter of the lt pulmonay artery >16mm 
C. Diameter of the decending Rt pulmonay artery >16mm 
D. Diameter of the decending Lt pulmonay artery >16mm 
Ans: C(REPEAT)

46. Necrotizing lymphadenitis is seen in? 


A. Kimura disease 
B. Kikuchi disease 
C. Hodgkin disease 
D. Castelma 
Ans: B Kikuchi disease (a type of Modified REPEAT) 
Reference:Harrison 17th ed and eMEDICINE 
Kikuchi disease, also called histiocytic necrotizing lymphadenitis or Kikuchi-Fujimoto disease 
The most common clinical manifestation of Kikuchi disease is cervical lymphadenopathy 
Kimura disease is a chronic inflammatory disorder of unknown etiology that most commonly presents as painless, unilateral cervical
lymphadenopathy or subcutaneous masses in the head or neck region. The disorder received its current name in 1948, when
Kimura et al2 noted the vascular component and referred to it as an "unusual granulation combined with hyperplastic changes in
lymphoid tissue." 
Lymph-Node Tuberculosis (Tuberculous Lymphadenitis) from Harrison 17th ed: 
The most common presentation of extrapulmonary tuberculosis (>40% of cases in the United States in recent series), lymph-node
disease is particularly frequent among HIV-infected patients. In the United States, children and women (particularly non-
Caucasians) also seem to be especially susceptible. Once caused mainly by M. bovis, tuberculous lymphadenitis is today due largely
to M. tuberculosis. Lymph-node tuberculosis presents as painless swelling of the lymph nodes, most commonly at posterior cervical
and supraclavicular sites (a condition historically referred to as scrofula). Lymph nodes are usually discrete and nontender in early
disease but may be inflamed and have a fistulous tract draining caseous material. Associated pulmonary disease is seen in >40% of
cases. Systemic symptoms are usually limited to HIV-infected patients. The diagnosis is established only by fine-needle aspiration
or surgical biopsy. AFB are seen in up to 50% of cases, cultures are positive in 70–80%, and histologic examination shows
granulomatous lesions. Among HIV-infected patients, granulomas usually are not seen. Differential diagnosis includes a variety of
infectious conditions, neoplastic diseases such as lymphomas or metastatic carcinomas, and rare disorders like Kikuchi disease
(necrotizing histiocytic lymphadenitis), Kimura's disease, and Castleman's disease. 

47. NARP syndrome is seen in? 


A. Mitochondrial function disorder 
B. Glycogen storage disorder 
C. Lysosomal storage disorder 
D. Lipid storage disorder 
Ans: A Mitochondrial disorder(REPEAT) 
48. A 65 yrs old lady presented with a swollen and painful knee. On examination, she was found to have grade III osteoarthritic
changes. What is the “BEST COURSE OF ACTION” ? 
A. Conservative management 
B. Arthroscopic washing 
C. Partial knee replacement 
D. Total knee replacement 
Ans: D Total knee replacement 
THIS QUESTION IS NOT A REPEAT FROM MAY AIIMS.... 
This question has been adequately discussed in the institute classes and the answer is TKR....Whatever be the case they are asking
the BEST course of action and so no point in going for a conservative approach... 
49. Causes of primary amenorrhoea are all except? 
A. Rokintasky syndrome 
B. Kallaman syndrome 
C. Sheehan syndrome D.Turner syndrome 
Ans: C Sheehan syndrome(REPEAT) 
50. Integrase inhibitor approved for treatment of HIV is? 
A. Raltegravir 
B. Indinavir 
C. Lopinavir 
D. Elvitegravir 
Ans: A Raltegravir 
Reference: Pasting a net reference since the texts don’t have thedrug....anyway Ralteravir and maraviroc are given in Sparsh
Pharmac 
Raltegravir (MK-0518, brand name Isentress) is an antiretroviral drug produced by Merck & Co., used to treat HIV infection.[1] It
received approval by theU.S. Food and Drug Administration (FDA) in October 2007, the first of a new class of HIV drugs, the
integrase inhibitors, to receive such approval.

1).Most common nerve injured in supracondylar fracture humerus? 


a. Median 
b. Radial 
c. Ulnar 
d. Anterior interosseus nerve 
Ans: D 
I would not like to discuss much on this question since it has already been discussed in detail previously...The Mnemonic for the
order of nerves injured in Supracondylar fracture is AMRU 
That is: 
Ant . Interosseus Nerve> Median > Radial >Ulnar 
2. Earliest symptom of GERD in an infant is? 
A. Respiratory distress answer 
B. Upper GI bleed 
C. regurgitation 
D.obstruction 
Ans: C REGURGITATION.... 
3. Basal metabolic rate is closely associated with? 
A. Lean body mass 
B. Body surface area 
C. Body mass index 
D. Body weight 
Ans: A Lean Body mass (REPEAT...shall not spend much time on repeats) 
4. A girl presented with recurrent occipital headache associated with ataxia and vertigo. Mother also has similar complaints. Most
probable diagnosis is? 
A. Vestibular neuronitis 
B. Basillar migraine 
C. TIA 
D. – 
Ans: b Basilar migraine(REPEAT) 
5. Drug of choice for central Diabetes Insipidus is? 
a. Desmopressin 
b. Leuperolide 
c. Thiazide diuretics 
d. – 
Ans: A Desmopressin 
6. A 32 year old mountaineer has a hematocrit of 70%. What is the possible explanation? 
A. Polycythemia with relative dehydration 
B. High altitude cerebral oedema 
C. High altitude pulmonary oedema 
D. Hemodilution 
Ans.A Polycythemia and relative dehydration 
7. Tetracycline is used in the prophylaxis of which of the following diseases? 
a. Cholera 
b. Brucellosis 
c. Leptospirosis 
d. Meningitis 
Ans: a Cholera 
8. A 7 month old child has bouts of cough ending with a whoop. What is the best way to confirm the diagnosis? 
A. Nasophayngeal swab 
B. Cough plate culture 
C. Tracheal aspirate 
D. – 
Ans: a Nasopharyngeal swab 
9. Aflatoxin is produced by? 
A. Aspergillus flavus 
B. Aspergillus niger 
C. Candida 
D. – 
Ans: a Aspergillus Flavus 
10. Most important prognostic factor in congenital diaphragmatic hernia? 
A. Pulmonary hypertension 
B. Timing of surgery 
C. Size of defect 
D. – 
Ans: A Pulmonary hypertension 
11. A surgeon removed the part of liver to the left of the falciform ligament. Which segments have been removed? 
A. 1 & 4a 
B. 2 & 3 
C. 1 & 4b 
D. 5 & 6 
Ans: B 2 & 3(REPEAT) 
12. Punnett square is used for 
a. Finding genotype of offspring 
b. Statistical analysis 
c. – 
d. – 
Ans:a Finding the genotype of the offspring 
13. Cavitation is seen in? 
A. Mycolplasma pneumonia 
B. Tuberculous pneumonia 
C. Streptococcal pneumonia 
D.Staphylococcus pneumonia 
Ans: D Staphylococcal Pneumonia(REPEAT) 
14. In PSUDOHYPOPARATHYROIDISM true is?(NEED FINETUNING OF STEM) 
a. Decreased cAMP 
b. Decreased IP3 
c. Increased gtpas activity/Gain of function mutation in GTPase 
d.--- 
Ans:a Decreased cAMP 
15. Difference between follicular carcinoma and follicular adenoma is? 
A. Vascular invasion 
B. Mitosis 
C. Nuclear pleomorphism 
D. Tubule formation 
Ans: a Vacular invasion (REPEAT) 
16. Antidepressant drug that can be used in nocturnal eneuresis? 
A. Imipramine 
B. Fluoxamine 
C. 
D. 
Ans: A Imipramine 
17. A female patient presented with depressed mood, loss of appetite and no interest in surroundings. There is associated
insomnia. The onset of depression was preceeded by a history of business loss. What is the line of management? 
A. No treatment is necessary as it is due to business loss 
B. SSRI is the most efficacious of the available drugs 
C. Start SSRI treatment based on side effect profile 
D. Combination therapy of 2 anti depressant drugs 
Ans:C Treatment is started based on the side effect profile 
18. An Infant is brought to casualty with reports of violent shaking by parents. Most characteristic injury is? 
A. Long bone fracture answer 
B. Ruptured spleen 
C. Subdural hematoma 
D. Skull bone fracture 
Ans: Subdural hematoma 
19. Gun powder on clothing can be visualized by? 
A. Magnifying lens 
B. UV rays 
C. Infrared rays 
D. Dye
Ans: B UV Rays (REPEAT)
20. Capsular antibody protection is seen in all except? 
A. Neisseria meningitidis 
B. Pneumococcus 
C. Bordetella pertussis 
D. Haemophilus influenza 
Ans: C Bordetella pertusis 
21. Signature fracture refers to? 
A. Depressed skull fracture 
B. Suture displacement fracture 
C. Contrecoup injury 
D. Fracture at foramen magnum 
Ans: Depressed fracture (REPEAT) I don’t think the question needs an explanation 
22. Which among the following is most frequently seen in anti phospholipid antibody syndrome?(Pls CORRECT Q) 
A. Beta 2 microglobulin antibody 
B. Anti nuclear antibody 
C. Anti centromere antibody 
D. Anti beta 2 glycoprotein antibody 
Ans: D Anti beta 2 GP Ab 
23. What factor is responsible for deciding whether an antibody will remain membrane bound or get secreted? 
A. RNA splicing 
B. Class switching 
C. Differential RNA processing 
D. Allelic exclusion 
Ans: C Differential RNA Processing(REPEAT) 
24. Blood Chimerism is associated with? 
A. Monochorionic monoamniotic twins 
B. Monochorionic diamniotic twins 
C. Singleton pregnancy 
D. Vanishing twin 
Ans: B Monochorionic diamniotic twins (no doubt if these were the options) 
25. In expectant management of placenta praevia, all are done except? 
A. Cervical encirclage 
B. Anti D 
C. Corticosteroids 
D. Blood transfusion 
Ans: a Cervical encirclage 
Reference: Williams Obstetrics 
The treatment of classical cervical incompetence is cerclage. The operation is performed to surgically reinforce 
the weak cervix by some type of purse-string suturing. Bleeding, uterine contractions, or ruptured membranes are usually
contraindications for cerclage. 
26. Which drug is not used during delivery in a woman with rheumatic heart disease ? 
A. Methylergometrine 
B. Carboprost 
C. Syntocin 
D. Misoprostol 
Ans: A Methylergometrine(REPEAT) 
27. Which is not an autoimmune disease? 
A. SLE 
B. Grave's disease 
C. Myasthenia gravis 
D. Sickle cell disease 
Ans: D Sickle Cell Disease(I don’t think an explanation is needed..Sickle cell ds is a hemoglobinopathy and not an autoimmune ds) 
28. All are true regarding selective estrogen receptor downregulator (SERD),fulvestrant except? 
A. Used for breast cancer 
B. Is a selective oestrogen antagonist 
C. Is slower acting, safer, LESS effective than SERM 
D. Given as once a month im dose 
Ans: C Its slower acting and less efficacious than SERM 
29. A farmer developed a swelling in the inguinal region which later ulcerated. What stain can be used to detect bipolar stained
organisms? 
A. Albert's stain 
B. Waysons stain 
C. Ziehl neelsen stain 
D. Nigrosin stain 
Ans: B Wayson stain(REPEAT) 
30. An 8 year old boy completed 8 out of 10 day course of cefaclor. Now he developed a generalized erythmatic rash which is mildly
pruritic and lymphadenopathy. Diagnosis is? 
A. Kawasaki disease 
B. Type 3 hypersensitivity 
C. Anaphylaxis 
D. Infectious mononucleosis 
Ans: B Type 3 Hypersensitivity(REPEAT) 
31. Rave drug is? 
A. Cannabis 
B. Cocaine 
C. Heroin 
D. Amphetamine 
Ans: D Amphetamine(MDMA) 
32. How to differentiate ASD from VSD in X-ray? 
A. Enlarged Left atrium 
B. Normal left atrium 
C. Pulmonary congestion 
D. Aortic shadow 
Ans: B Normal LA size 
33. Regarding an imbecile, all are true except? 
A. IQ is 50-60 B. Intellectual capacity equivalent to a child of 3-7 years of age 
C. Not able to take care of themselves 
D. Condition is congenital or acquired at an early age 
Ans: A IQ is 50-60 
34. Which is not true regarding diet modification recommended in high cardiovascular risk group? 
A. Cholesterol less then 100 mg/1000kcal/day 
B. Avoid alcohol 
C. Fat intake 10% of total calories 
D. Salt limitation to less than 5 gm 
Ans: C Fat intake <10% of total calories 
35. Maintenance dose of which of the following drugs is used worldwide for opioid dependence? 
A. Naltrexone 
B. Methadone 
C. lmipramine 
D. Disulfiram 
Ans: B Methadone 
36. Best test/Gold standard test for assesing HCG function/action? 
A. Radioimmunoassay 
B. ELISA 
C. Latex test 
D. Bioassay 
Ans: D Bioassay(OPEN TO SCRUTINY......CHALLENGING REFERENCES INVITED) 
37. Vitamin K is involved in the posttranswerlational modification of? 
A. Glutamate 
B. Aspartate 
C. -- 
D. – 
Ans: A Glutamate(REPEAT) 
38. Spinal anaesthesia is given at which level? 
A. L1-2 
B. L2-4 
C. S1 
D. Midline thoracic segments 
Ans: B L2-4 
39. a child presented with mild fever little breathlessness..... was treated and she improved over 4 days and later deteriorated
again with fever and more breathlessness. x ray showed hyperlucency. diagnosis? 
1.bronchiolitis obliterans 
2.alveolar proteinosois 
3.bronchitis 4. AsthmA 
Ans: a bronchiolitis obliterans(REPEAT) 
40. Which of the following passes through foramen magnum? 
A. Internal Carotid Artery 
B. Sympathetic chain 
C. Hypoglossal Nerve 
D. Vertebral Artery 
Ans: D Vertebral artery(REPEAT) 
41. McKeon's theory on reduced prevalence of TB? 
A. Increased awareness and knowledge 
B. Medical advancement answer 
C. Behavioural modification 
D. Social and environmental factor 
Ans: D Social and environmental factor 

42. Which among the following is not a cause of fasting hypoglycemia? 


A. Glucagon excess 
B. Glucose 6 phospatase deficiency 
C. Ureamia 
D. Glycogen synthase deficiency 
Ans: A Glucagon excess 
43. Mineralocorticoid receptor is not present in? 
A. Liver 
B. Colon 
C. Hippocampus 
D. Kidney 
Ans:Liver(REPEAT) 
44. Prolonged treatment with INH leads to deficiency of? 
A. Pyridoxine 
B. Thiamine 
C. Pantothenic acid 
D. Niacin 
Ans: A Pyridoxine 
45. Which is the most reliable objective sign of identifying pulmonary plethora in chest X-ray? 
A. Diameter of the main pulmonay artery >16mm 
B. Diameter of the lt pulmonay artery >16mm 
C. Diameter of the decending Rt pulmonay artery >16mm 
D. Diameter of the decending Lt pulmonay artery >16mm 
Ans: C(REPEAT) 
46. Necrotizing lymphadenitis is seen in? 
A. Kimura disease 
B. Kikuchi disease 
C. Hodgkin disease 
D. Castelma 
Ans: B Kikuchi disease (a type of Modified REPEAT) 
47. NARP syndrome is seen in? 
A. Mitochondrial function disorder 
B. Glycogen storage disorder 
C. Lysosomal storage disorder 
D. Lipid storage disorder 
Ans: A Mitochondrial disorder(REPEAT) 
48. A 65 yrs old lady presented with a swollen and painful knee. On examination, she was found to have grade III osteoarthritic
changes. What is the “BEST COURSE OF ACTION” ? 
A. Conservative management 
B. Arthroscopic washing 
C. Partial knee replacement 
D. Total knee replacement 
Ans: D Total knee replacement 
49. Causes of primary amenorrhoea are all except? 
A. Rokintasky syndrome 
B. Kallaman syndrome 
C. Sheehan syndrome D.Turner syndrome 
Ans: C Sheehan syndrome(REPEAT) 
50. Integrase inhibitor approved for treatment of HIV is? 
A. Raltegravir 
B. Indinavir 
C. Lopinavir 
D. Elvitegravir 
Ans: A Raltegravir

51. Deoxygenated blood is not seen in 


a. Pulmonary artery 
b. Umbilical artery 
c. Umbilical vein 
d. Renal vein 
Ans: C Umbilical vein(No explanation needed) 
52. All of the following are pneumatic bones except? 
A. Frontal 
B. Ethmoid 
C. Mandible 
D. Maxilla 
Ans: C Mandible (No explanation needed) 
53. Which of the following is not a contraindication for pregnancy? 
A. WPW syndrome 
B. Pulmonary hypertension 
C. Eisenmenger syndrome 
D. Marfan syndrome with aortic root dilatation 
Ans: A WPW syndrome(REPEAT) 
54. Which of the following antihypertensive drugs is contraindicated in a patient on Lithium in order to prevent toxicity? 
A. Clonidine 
B. Beta blockers 
C. Calcium channel blockers 
D. Diuretics 
Ans: D Diuretics 
Reference: goodman and gilman 
Most of the renal tubular reabsorption of Li+ occurs in the proximal tubule. Nevertheless, Li+ retention can be increased by any
diuretic that leads to depletion of Na+, particularly the thiazides (see Chapter 28) (Siegel et al., 1998). Renal excretion can be
increased by administration of osmotic diuretics, acetazolamide, or aminophylline, although they are of little help in the
management of Li+ intoxication. Triamterene may increase excretion of Li+, suggesting that some reabsorption of the ion may
occur in the distal nephron. However, spironolactone does not increase the excretion of Li+. Some nonsteroidal antiinflammatory
agents can facilitate renal proximal tubular resorption of Li+ and thereby increase concentrations in plasma to toxic levels 
55. Superior vena caval syndrome is most commonly caused by? 
A. Lymphoma 
B. Small cell lung ca 
C. Non small cell lung ca 
D. Secondary tumours 
Ans: B Small cell carcinoma(REPEAT)
56. Which of the following is not an adverse effect of thalidomide? 
A. Diarrhoea 
B. Teratogenicity 
C. DVT 
D. Peripheral neuropathy 
Ans: A Diarrhea 
Reference: Goodman and Gilman 
The most common adverse effects reported in cancer patients are sedation and constipation (Franks et al., 2004), while the most
serious one is treatment-emergent peripheral sensory neuropathy, which occurs in 10% to 30% of patients with MM or other
malignancies in a dose- and time-dependent manner (Richardson et al., 2004). Thalidomide-related neuropathy is an asymmetric,
painful, peripheral paresthesia with sensory loss, commonly presenting with numbness of toes and feet, muscle cramps, weakness,
signs of pyramidal tract involvement, and carpal tunnel syndrome. The incidence of peripheral neuropathy increases with higher
cumulative doses of thalidomide, especially in elderly patients. Although clinical improvement typically occurs upon prompt drug
discontinuation, long-standing residual sensory loss can occur. Particular caution should be applied in cancer patients with
preexisting neuropathy (e.g., related to diabetes) or prior exposure to drugs that can cause peripheral neuropathy (e.g., vinca
alkaloids or bortezomib), especially since there has been little progress in defining effective strategies to alleviate neuropathic
symptoms. An increasing incidence of thromboembolic events in thalidomide-treated patients has been reported, but mostly in the
context of thalidomide combinations with other drugs, including steroids and particularly anthracycline-based chemotherapy
(Zangari et al., 2001), and with very low incidence with single-agent thalidomide treatment. 
57. Blount’s disease is: 
A. Genu valgum 
B. Genu varum 
C. Genu recurvatum 
D. Menisceal injury 
Ans: B Genu varum 
Reference:eMEDICINE(don’t have the orthobook with me right now) 
Blount disease is an uncommon growth disorder characterized by disordered ossification of the medial aspect of the proximal tibial
physis, epiphysis, and metaphysis. This progressive deformity is manifested by varus angulation and internal rotation of the tibia in
the proximal metaphyseal region immediately below the knee 
58. A teenaged girl complains of pain in knee on climbing stairs and on getting up after sitting for a long time. What is the probable
diagnosis? 
A. Chondromalacia patellae 
B. Plica syndrome 
C. Bipartite patella 
D. Patello-femoral osteoarthritis 
Ans: A Chondromalacia patellae 
Reference: Current Orthopedic Diagnosis and management. 
Bilateral knee pain and stiffness in a teenage girl is suggestive of a few conditions of which Chondromalacia patellae is one. 
59. Which of the following is not included in parenteral nutrition? 
A. Fat 
B. Carbohydrate 
C. Fibre 
D. Micronutrients 
Ans: C Fibre 
60. Sparrow marks are seen in? 
A. Gunshot injuries 
B. Stab injury of face 
C. Vitriolage 
D. Windshield glass injury 
Ans: D Winshield glass injury 
Reference: Forensic pathology by David.J.Williams 
“The occupants of the first row seats in a motor vehicle accident may demonstrate facial injuries due to contact with windshield
glass called sparrow’s foot.”

1. Amphotericin B causes deficiency of? 


A. Na 
B. Ca 
C. K 
D. Mg 
Ans: C Potassium 
Reference: Goodman and Gilman 
Renal tubular acidosis and renal wasting of K+ and Mg2+ also may be seen during and for several weeks after therapy.
Supplemental K+ is required in one-third of patients on prolonged therapy. 
62. All are seen in injury to common peroneal nerve except? 
A. Loss of sensation over sole 
B. Foot drop 
C. Injury to neck of fibula 
D. Loss of dorsiflexion of toe 
Ans: A loss of sensation over the sole(No explanation needed) 
63. Cause of premature death in schizophrenia? 
A. Homicide 
B. Suicide 
C. Toxicity of antipsychotic drug 
D. Hospital acquired infection 
Ans:B Suicide 
Reference:Kaplan and saddock(A STAIGHT LINE TO LINE FROM THE TEXT) 
Suicide 

Suicide is the single leading cause of premature death among people with schizophrenia. Suicide attempts are made by 20 to 50
percent of the patients, with long-term rates of suicide estimated to be 10 to 13 percent. These numbers reflect an approximately
20-fold increase over the suicide rate in the general population. Often, suicide in schizophrenia seems to occur “out of the
blue,” without prior warnings or expressions of verbal intent. The most important factor is the presence of a major depressive
episode. Epidemiological studies indicate that up to 80 percent of schizophrenia patients may have a major depressive episode at
some time in their lives. Some data suggest that those patients with the best prognosis (few negative symptoms, preservation of
capacity to experience affects, better abstract thinking) can paradoxically also be at highest risk for suicide. The profile of the
patient at greatest risk is a young man 
64. Epileptic potential is present in 
A. Desflurane 
B. Halothane 
C. Sevoflurane 
D. Ether 
Ans: Sevoflurane(No explanation needed) 
65. Which of the following anesthetic drugs is contraindicated in a patient with hypertension? 
A. Ketamine 
B. Propofol 
C. Etomidate 
D. Diazepam 
Ans: Ketamine(Ketamine is known to increase the BP...no explanation needed)

66. All of the following decrease bone resorption in osteoporosis except? 


A. Alendronate 
B. Etidronate 
C. Strontium 
D. Teriparatide 
Ans: D Teripartide 

67. 26-Ondoni cells and Haller cells are associated with the following structures respectively? 
A. Optic nerve and Orbital floor answer 
B. Optic nerve and Internal carotid artery 
C. Internal carotid artery and Optic nerve 
D. Orbital floor and Internal carotid artery 
Ans: Optic nerve and Orbital floor(REPEAT) 

68. 27-Pain sensation from the ethmoid sinus is carried by : 


A. Frontal nerve 
B. Lacrimal nerve 
C. Nasociliary nerve 
D. Infraorbital nerve 
Ans: C Nasociliary nerve which divides into the ethmoidal branches to supply the ethmoidal sinus 

69. 28-Which among the following is not used to treat alcohol dependence? 
A. Flumazenil 
B. Acamprosate 
C. Naltrexone 
D. Disulfiram 
Ans: A Flumazenil(a MODIFIED REPEAT......last time this question came with Diazepam in the options) 
70. A 40yr old patient has a single kidney with an exophytic mass of 4 cm size at it’s lower pole. Which among the following is the
best course of action? 
A. Partial nephrectomy 
B. Radical nephrectomy with dialysis 
C. Radical nephrectomy with immediate renal transwerplant 
D. Observation 

Ans: A Partial nephrectomy since it is the current method of choice for tumors less than or equal to 4cm and at poles.

71. 29-Which among the following is the most common fungal infection seen in immuno competent patients? 
A. Aspergillus 
B. Candida 
C. Cryptococcus 
D. Mucor 

Ans: A Aspergillus 
Reference: Harrison 17th ed 
The required size of the infecting inoculum is uncertain; however, only intense exposures (e.g., during construction work, handling
of moldy bark or hay, or composting) are sufficient to cause disease in healthy immunocompetent individuals. 
An increasing incidence of invasive aspergillosis in medical intensive care units suggests that, in patients who are not
immunocompromised, temporary abrogation of protective responses as a result of glucocorticoid use or a general anti-inflammatory
state is a significant risk factor. 

72. All are seen in Argyl Robertson pupil except? 


A. Near reflex normal 
B. Direct reflex absent 
C. Consensual reflex normal 
D. Vision normal 

Ans: C Consensual reflex is normal 


Argyll Robertson pupil 
This is caused by neurosyphilis and is characterised by the 
following: 
• Involvement is usually bilateral but asymmetrical 
• The pupils arc small and irregular. 
• Light-near dissociation. • The pupils are very difficult to dilate. 

73. 31-A 5 year old boy while having dinner suddenly becomes aphonic and is brought to the casulty for the complaint of
respiratory distress. What should be the appropriate management? 
A. Cricothyroidotomy 
B. Emergency tracheostomy 
C. Humidified oxygen 
D. Heimlich maneuver 

Ans: D Heimlich manoeveure(please read the explanation carefully) 

This is the AHA guidelines for management of any conscious choking patient....The question in the exam had a conscious,aphonic
patient in respiratory distress...Now how to manage.....Most of the discussions in the forum give other answer feeling that Heimlich
has to be performed at the site only...lets C the guidelines and find the answer... 

1. Recognise signs of choking 


2. Call for help 
3. Use Heimlich manoeveure until the patient recovers OR LOSES CONSCIOSNESS 
4. Place the victim in a supine position ..open the mouth and perform a finger sweep 
5. Open the airway and attempt to ventilate 
6. If unsuccessful give upto 5 heimlich manoeveures 
7. Repeat 5 and 6 
8. If expert has arrived he has to perform a laryngoscopy and try to remove the foreign body if not already removed 
9. FAILING ALL ATTEMPTS ONLY CRICOTHYOTOMY SHOULD BE PERFORMED. 

74.Which among the following is a branch from the trunk of brachial plexus? 
A. Subscapular nerve 
B. Long thoracic nerve 
C. Anterior thoracic nerve 
D. Nerve to subclavius 

Ans:D Nerve to subclavius.......If i am sure suprascapular nerve was not at all there in the options....so the answer is clear.... 

75.Orthotolidine test is used for detecting: 


A. Chlorine 
B. Nitrites 
C. Nitrates 
D. Ammonia 
Ans: A Chlorine(No explanation needed) 
76. Which among the following is the most common tumour associated with neurofibromatosis in a child? 
A. Juvenile myelomonocytic leukemia 
B.Acute lymphoblastic leukemia 
C. Acute monocytic leukemia 
D. Acute myeloid leukemia 

Ans: Juvenile myelomonocytic leukemia 


Reference: Nelson 18th ed 
Juvenile Chronic Myelogenous Leukemia 
Juvenile chronic myelogenous leukemia (JCML), also known as juvenile myelomonocytic leukemia, is a clonal proliferation of
hematopoietic stem cells that typically affects children <2 yr of age. Patients with this disease do not have the Philadelphia
chromosome that is characteristic of CML. Patients with JCML present with rashes, lymphadenopathy, and splenomegaly. Analysis of
the peripheral blood often shows an elevated leukocyte count and also may show thrombocytopenia and the presence of
erythroblasts. The bone marrow shows a myelodysplastic pattern, with blasts accounting for <30% of cells. No distinctive
cytogenetic abnormalities are seen. JCML is rare, constituting <2% of all cases of childhood leukemia. Therapeutic reports are
largely anecdotal. Patients with neurofibromatosis type 1 have a predilection for this type of leukemia. Stem cell transplantation
offers the best opportunity for cure, but much less so than for classic CML.

77. Diaphragm develops from all except: 


A. Septum transwerversum 
B. Dorsal mesocardium 
C. Pleuroperitoneal membrane 
D. Cervical myotomes 

Ans: D Cervical myotomes 


Reference: Langman Embryology gives text confirming the options A/D ....now regarding options B and D is the
confusion....Langman says the nerve supply alone develops from the level of the cervical myotomes but not the diaphragm....so i
prefer the answer cervical myotomes> dorsal mesocardium 

78. Erythema nodosum is seen in all except: 


A. Pregnancy 
B. Tuberculosis 
C. SLE 
D. Chronic pancreatitis 

Ans: D Chronic pancreatitis 


Causes 
Currently, the most common cause of erythema nodosum is streptococcal infection in children and streptococcal infection and
sarcoidosis in adults.3 Numerous other causes have been reported.4 The causes reported most often in the literature are as
follows: 
• Bacterial infections: Streptococcal infections are one of the most common causes of erythema nodosum. Tuberculosis was an
important cause in the past, but it has decreased dramatically as a cause for erythema nodosum; however, it still must be
excluded, especially in developing countries.5,6 Yersinia enterocolitica is a gram-negative bacillus that causes acute diarrhea and
abdominal pain; it is a common cause of erythema nodosum in France and Finland.7,8,9 Mycoplasma pneumoniae infection may
cause erythema nodosum. Erythema nodosum leprosum clinically resembles erythema nodosum, but the histologic picture is that of
leukocytoclastic vasculitis. Lymphogranuloma venereum may cause erythema nodosum. Salmonella infection may cause erythema
nodosum. Campylobacter infection may cause erythema nodosum. 
• Fungal infections: Coccidioidomycosis (San Joaquin Valley fever) is the most common cause of erythema nodosum in the
American Southwest. In approximately 4% of males and 10% of females, the primary fungal infection (which may be asymptomatic
or involve symptoms of upper respiratory infection) is followed by the development of erythema nodosum. Lesions appear 3 days to
3 weeks after the end of the fever caused by the fungal infection. Histoplasmosis may cause erythema nodosum. Blastomycosis
may cause erythema nodosum. 
• Drugs: Sulfonamides and halide agents are an important cause of erythema nodosum. Drugs more recently described to cause
erythema nodosum include gold and sulfonylureas. Oral contraceptive pills are implicated in an increasing number of reports. 
• Enteropathies: Ulcerative colitis and Crohn disease may trigger erythema nodosum. Erythema nodosum associated with
enteropathies correlates with flares of the disease. The mean duration of chronic ulcerative colitis before the onset of erythema
nodosum is 5 years, and erythema nodosum is controlled with adequate therapy of the colitis. Erythema nodosum is the most
frequent dermatologic symptom ininflammatory bowel diseases, and it is strongly associated with Crohn disease.10 
• Hodgkin disease and lymphoma: Erythema nodosum associated with non-Hodgkin lymphoma may precede the diagnosis of
lymphoma by months. Reports of erythema nodosum preceding the onset of acute myelogenous leukemia have been published.11 
• Sarcoidosis: The most common cutaneous manifestation of sarcoidosis is erythema nodosum. A characteristic form of acute
sarcoidosis involves the association of erythema nodosum, hilar lymphadenopathy, fever, arthritis, and uveitis, which has been
termed Löfgren syndrome. This presentation has a good prognosis, with complete resolution within several months in most
patients. HLA-DRB1*03 is associated with Löfgren syndrome. Most DRB1*03-positive patients have resolution of their symptoms
within 2 years; however, nearly half of DRB1*03-negative patients have an unremitting course.12 
• Behçet disease (associated with erythema nodosum) 
• Pregnancy: Some patients develop erythema nodosum during pregnancy, most frequently during the second trimester. Repeated
episodes occur with subsequent pregnancies or with the use of oral contraceptives. 

79. Posterior relations of head of pancreas are all except? 


A. Common bile duct 
B. First part of duodenum 
C. Aorta 
D. Inferior vena cava 
Ans: B First part of Duodenum(NO Explanation needed) 
80. Multiple sebaceous cysts seen in: 
A. Gardner’s syndrome 
B. Turcot syndrome 
C. Muir Torre syndrome 
D. Cowden syndrome 
Ans:C Muir torre syndrome 
Reference:Sabiston 18th ed table in chapter of GI neoplasia 
Muir-Torre variant: sebaceous adenomas, keratoacanthomas, sebaceous epitheliomas, and basal cell epitheliomas 

81. Which among the following is the hallmark of acute inflammation? 


A. Vasoconstriction 
B. Stasis 
C. Vasodilation and increase in permeability 
D. Leukocyte margination 
Ans: C Vasodilatation and increased vascular permeability(REPEAT)

82. True about epidural opioids are all except? 


A. Acts on dorsal horn substantia gelatinosa 
B. Can cause Itching 
C. Function of the intestines are not affected 
D. Can cause respiratory depression 

Ans: C Function of the intestine is not affected 

Reference: Pharmacology By KATZUNG(THE QUESTION HAS BEEN TAKEN LINE TO LINE FROM TEXT---READ BELOW) 

Because of their direct action on the superficial neurons of the spinal cord dorsal horn, opioids can also be used as regional
analgesics by administration into the epidural or subarachnoid spaces of the spinal column. A number of studies have demonstrated
that long-lasting analgesia with minimal adverse effects can be achieved by epidural administration of 3–5 mg of morphine,
followed by slow infusion through a catheter placed in the epidural space. It was initially assumed that the epidural application of
opioids might selectively produce analgesia without impairment of motor, autonomic, or sensory functions other than pain.
However, respiratory depression can occur after the drug is injected into the epidural space and may require reversal with
naloxone. Effects such as pruritus and nausea and vomiting are common after epidural and subarachnoid administration of opioids
and may also be reversed with naloxone if necessary. Currently, the epidural route is favored because adverse effects are less
common.

83. All are true about neuronal tumors except? 


A. 90% are malignant 
B. 95% occur in the abdomen 
C. They secrete catecholamines 
D. They arise from sympathetic ganglions 

Ans: A 90% are malignant...as per the rule of 10...only 10% are malignant 
Reference:Harrison 17th ed 
Epidemiology 
Pheochromocytoma is estimated to occur in 2–8 out of 1 million persons per year, and about 0.1% of hypertensive patients harbor
a pheochromocytoma. Autopsy series reveal prevalence figures of 0.2%. The mean age at diagnosis is about 40 years, although the
tumors can occur from early childhood until late in life. The "rule of tens" for pheochromocytomas states that about 10% are
bilateral, 10% are extraadrenal, and 10% are malignant. However, these percentages are higher in the inherited syndromes.

1. Most common nerve injured in supracondylar fracture humerus? 


a. Median 
b. Radial 
c. Ulnar 
d. Anterior interosseus nerve 
Ans: D 
I would not like to discuss much on this question since it has already been discussed in detail previously...The Mnemonic
for the order of nerves injured in Supracondylar fracture is AMRU 
That is: 
Ant . Interosseus Nerve> Median > Radial >Ulnar 

2. Earliest symptom of GERD in an infant is? 


A. Respiratory distress answer 
B. Upper GI bleed 
C. regurgitation 
D.obstruction 

Ans: C REGURGITATION.... 
Reference from Nelson Pediatrics 
Infantile reflux manifests more often with regurgitation (especially postprandially), signs of esophagitis (irritability,
arching, choking, gagging, feeding aversion), and resulting failure to thrive; symptoms resolve spontaneously in the
majority by 12–24 mo. Older children, in contrast, may have regurgitation during the preschool years; complaints of
abdominal and chest pain supervene in later childhood and adolescence. Occasional children present with neck
contortions (arching, turning of head) designated Sandifer syndrome. The respiratory presentations are also age
dependent: GERD in infants may manifest as obstructive apnea or as stridor or lower airway disease in which reflux
complicates primary airway disease such as laryngomalacia or bronchopulmonary dysplasia. Otitis media, sinusitis,
lymphoid hyperplasia, hoarseness, vocal cord nodules, and laryngeal edema have all been associated with GERD. In
contrast, airway manifestations in older children are more frequently related to asthma or to otolaryngologic disease such
as laryngitis or sinusitis. 
3. Basal metabolic rate is closely associated with? 
A. Lean body mass 
B. Body surface area 
C. Body mass index 
D. Body weight 
Ans: A Lean Body mass (REPEAT...shall not spend much time on repeats) 
4. A girl presented with recurrent occipital headache associated with ataxia and vertigo. Mother also has similar
complaints. Most probable diagnosis is? 
A. Vestibular neuronitis 
B. Basillar migraine 
C. TIA 
D. – 
Ans: b Basilar migraine(REPEAT) 

5. Drug of choice for central Diabetes Insipidus is? 


a. Desmopressin 
b. Leuperolide 
c. Thiazide diuretics 
d. – 
Ans: A Desmopressin 
Reference: Harrispn 17th ed 
The signs and symptoms of uncomplicated pituitary DI can be eliminated completely by treatment with desmopressin
(DDAVP), a synthetic analogue of AVP (Fig. 334-1). It acts selectively at V2 receptors to increase urine concentration and
decrease urine flow in a dose-dependent manner (Fig. 334-4). It is also more resistant to degradation than AVP and has
a three- to fourfold longer duration of action. Desmopressin (DDAVP) can be given by IV or SC injection, nasal inhalation,
or oral tablet. 
6. A 32 year old mountaineer has a hematocrit of 70%. What is the possible explanation? 
A. Polycythemia with relative dehydration 
B. High altitude cerebral oedema 
C. High altitude pulmonary oedema 
D. Hemodilution 

Ans.A Polycythemia and relative dehydration 

Reference:Harrison 17th ed 


Polycythemia can be spurious (related to a decrease in plasma volume; Gaisbock's syndrome), primary, or secondary in
origin. The secondary causes are all associated with increases in EPO levels: either a physiologically adapted appropriate
elevation based on tissue hypoxia (lung disease, high altitude, CO poisoning, high-affinity hemoglobinopathy) or an
abnormal overproduction (renal cysts, renal artery stenosis, tumors with ectopic EPO production). A rare familial form of
polycythemia is associated with normal EPO levels but hyperresponsive EPO receptors due to mutations 

7. Tetracycline is used in the prophylaxis of which of the following diseases? 


a. Cholera 
b. Brucellosis 
c. Leptospirosis 
d. Meningitis 
Ans: a Cholera 
Reference:Goodman Gilman and Harrison 17th ed 
Doxycycline (300 mg as a single dose) is effective in reducing stool volume and eradicating Vibrio cholerae from the stool
within 48 hours. Antimicrobial agents, however, are not substitutes for fluid and electrolyte replacement in this disease.
In addition, some strains of V. cholerae are resistant to tetracyclines. 
8. A 7 month old child has bouts of cough ending with a whoop. What is the best way to confirm the diagnosis? 
A. Nasophayngeal swab 
B. Cough plate culture 
C. Tracheal aspirate 
D. – 

Ans: a Nasopharyngeal swab 


Harrison 17th ed 

The diagnosis is Pertusis and we have to find the best specimen to confirm the diagnosis. 

Culture of nasopharyngeal secretions remains the gold standard of diagnosis, although DNA detection by polymerase
chain reaction (PCR) is replacing culture in many laboratories because of increased sensitivity and quicker results. The
best specimen is collected by nasopharyngeal aspiration, in which a fine flexible plastic catheter attached to a 10-mL
syringe is passed into the nasopharynx and withdrawn while gentle suction is applied. 

9. Aflatoxin is produced by? 


A. Aspergillus flavus 
B. Aspergillus niger 
C. Candida 
D. – 

Ans: a Aspergillus Flavus 

MYCOTOXINS 
Many fungi produce poisonous substances called mycotoxins that can cause acute or chronic intoxication and damage.
The mycotoxins are secondary metabolites, and their effects are not dependent on fungal infection or viability. A variety
of mycotoxins are produced by mushrooms (eg, amanita species), and their ingestion results in a dose-related disease
called mycetismus. Cooking has little effect on the potency of these toxins, which may cause severe or fatal damage to
the liver and kidney. Other fungi produce mutagenic and carcinogenic compounds that can be extremely toxic for
experimental animals. One of the most potent is aflatoxin, which is elaborated by Aspergillus flavus and related molds
and is a frequent contaminant of peanuts, corn, grains, and other foods. 

10. Most important prognostic factor in congenital diaphragmatic hernia? 


A. Pulmonary hypertension 
B. Timing of surgery 
C. Size of defect 
D. – 

Ans: A Pulmonary hypertension 

Reference: Nelson 18th ed 

Overall survival of live-born infants is 67%. The incidence of spontaneous fetal demise with CDH diagnosis is 7–10%.
Relative predictors of a poor prognosis include an associated major anomaly, symptoms before 24 hr of age, severe
pulmonary hypoplasia, herniation to the contralateral lung, and the need for ECMO. 
Serious sequelae include pulmonary function changes, neurodevelopmental delays, and growth retardation. Pulmonary
problems continue to be a source of morbidity for long-term survivors of CDH. Children receiving CDH repair studied at
6–11 yr of age demonstrate significant decreases in forced expiratory flow at 50% of vital capacity and decreased peak
expiratory flow. Both obstructive and restrictive patterns can occur. Those without severe pulmonary hypertension and
barotrauma do the best. Those at highest risk include children who required ECMO and patch repair, but the data clearly
show that non-ECMO CDH survivors also require frequent attention to pulmonary issues. At discharge, up to 20% of
infants require oxygen, but only 1–2% require it past 1 yr of age. 
11. A surgeon removed the part of liver to the left of the falciform ligament. Which segments have been removed? 
A. 1 & 4a 
B. 2 & 3 
C. 1 & 4b 
D. 5 & 6 
Ans: B 2 & 3(REPEAT) 

Reference: Sabiston 18th ed 


Resection of segments II and III is a commonly performed sublobar resection and is often referred to as a left lateral
segmentectomy and left lateral sectionectomy or left lobectomy.Left lobe is that part of liver to the left of the falciform
ligament. 

12. Punnett square is used for 


a. Finding genotype of offspring 
b. Statistical analysis 
c. – 
d. – 
Ans:a Finding the genotype of the offspring 
Reference: The Tennessee Gateway Science 
Punnett square is a Diagram used to identify possible combinations of recessive and dominant alleles in OFFSPRING.To
create a punnette square divide a suare into 4 parts and write the letters that represent the alleles of one parent on top
of the suare and the alleles of the other parent on the side of the square..Just like we make a 2X2 table for statistics
questions.Combine the allele of one parent with the other and work out the 4 possible combinations.The punnett square
does not give the exact information about the offspring but instead gives the probability. 

Have a look at this diagram...its a simple concept that we all have been practising since childhood but may not have been
aware of the name of this method... 

13. Cavitation is seen in? 


A. Mycolplasma pneumonia 
B. Tuberculous pneumonia 
C. Streptococcal pneumonia 
D.Staphylococcus pneumonia 

Ans: D Staphylococcal Pneumonia(REPEAT) 

14. In PSUDOHYPOPARATHYROIDISM true is?(NEED FINETUNING OF STEM) 


a. Decreased cAMP 
b. Decreased IP3 
c. Increased gtpas activity/Gain of function mutation in GTPase 
d.--- 

Ans:a Decreased cAMP 

Reference: Harrisson 17th ed 


Individuals with Pseudohypoparathyroidism 1, the most common of the disorders, show a deficient urinary cyclic AMP
response to administration of exogenous PTH. Patients with PHP-I are divided into type a, with AHO and reduced amounts
of Gs in in vitro assays with erythrocytes, and type b, lacking AHO and with normal amounts of Gs in erythrocytes. There
is a third type (PHP-Ic, reported in a few patients) that differs from PHP-Ia only in having normal erythrocyte levels of Gs
despite having AHO, hypocalcemia, and decreased urinary cyclic AMP responses to PTH (presumably with a post-Gs
defect in adenyl cyclase stimulation). 
15. Difference between follicular carcinoma and follicular adenoma is? 
A. Vascular invasion 
B. Mitosis 
C. Nuclear pleomorphism 
D. Tubule formation 
Ans: a Vacular invasion (REPEAT) 
16. Antidepressant drug that can be used in nocturnal eneuresis? 
A. Imipramine 
B. Fluoxamine 
C. 
D. 
Ans: A Imipramine 
Reference: Kaplan Saddock Psychiatry 
Imipramine (Tofranil) is efficacious and has been approved for use in treating childhood enuresis, primarily on a short-
term basis. Initially, up to 30 percent of patients with enuresis stay dry, and up to 85 percent wet less frequently than
before treatment. The success often does not last, however, and tolerance can develop after 6 weeks of therapy. Once
the drug is discontinued, relapse and enuresis at former frequencies usually occur within a few months. The drug's
adverse effects, which include cardiotoxicity, are also a serious problem.

17. A female patient presented with depressed mood, loss of appetite and no interest in surroundings. There is associated
insomnia. The onset of depression was preceeded by a history of business loss. What is the line of management? 
A. No treatment is necessary as it is due to business loss 
B. SSRI is the most efficacious of the available drugs 
C. Start SSRI treatment based on side effect profile 
D. Combination therapy of 2 anti depressant drugs 

Ans:C Treatment is started based on the side effect profile 

Reference:Kaplan and Saddock 

Initial Medication Selection 


The available antidepressants do not differ in overall efficacy, speed of response, or long-term effectiveness.
Antidepressants, however, do differ in their pharmacology, drug–drug interactions, short- and long-term side effects,
likelihood of discontinuation symptoms, and ease of dose adjustment. Failure to tolerate or to respond to one medication
does not imply that other medications will also fail. Selection of the initial treatment depends on the chronicity of the
condition, course of illness (a recurrent or chronic course is associated with increased likelihood of subsequent depressive
symptoms without treatment), family history of illness and treatment response, symptom severity, concurrent general
medical or other psychiatric conditions, prior treatment responses to other acute phase treatments, potential drug–drug
interactions, and patient preference. In general, approximately 45 to 60 percent of all outpatients with uncomplicated
(i.e., minimal psychiatric and general medical comorbidity), nonchronic, nonpsychotic major depressive disorder who
begin treatment with medication respond (i.e., achieve at least a 50 percent reduction in baseline symptoms); however,
only 35 to 50 percent achieve remission (i.e., the virtual absence of depressive symptoms). 

18. An Infant is brought to casualty with reports of violent shaking by parents. Most characteristic injury is? 
A. Long bone fracture answer 
B. Ruptured spleen 
C. Subdural hematoma 
D. Skull bone fracture 

Ans: Subdural hematoma 

Reference: Forensic Medicine By Reddy 

Under the Topic Battered Baby syndrome Reddy talks about SUBDURAL HEMATOMA being the MOST CHARACTERISTIC
FEATURE of violent shaking of an infant by the Parent...Its called INFANTILE WHIPLASH SYNDROME....In Battered baby
syndrome multiple Long bone fractures at various stages of healing may be seen and not in infantile whiplash
syndrome.. 

19. Gun powder on clothing can be visualized by? 


A. Magnifying lens 
B. UV rays 
C. Infrared rays 
D. Dye 
Ans: B UV Rays (REPEAT) 
20. Capsular antibody protection is seen in all except? 
A. Neisseria meningitidis 
B. Pneumococcus 
C. Bordetella pertussis 
D. Haemophilus influenza 

Ans: C Bordetella pertusis 

Reference: Harrison 17th ed 

Pathogenetic factors in Bordetella : 


B. pertussis produces a wide array of toxins and biologically active products that are important in its pathogenesis and in
immunity. Most of these virulence factors are under the control of a single genetic locus that regulates their production,
resulting in antigenic modulation and phase variation. Although these processes occur both in vitro and in vivo, their
importance in the pathobiology of the organism is unknown; they may play a role in intracellular persistence and person-
to-person spread. The organism's most important virulence factor is pertussis toxin, which is composed of a B oligomer–
binding subunit and an enzymatically active A protomer that ADP-ribosylates a guanine nucleotide-binding regulatory
protein (G protein) in target cells, producing a variety of biologic effects. Pertussis toxin has important mitogenic activity,
affects the circulation of lymphocytes, and serves as an adhesin for bacterial binding to respiratory ciliated cells. Other
important virulence factors and adhesins are filamentous hemagglutinin, a component of the cell wall, and pertactin, an
outer-membrane protein. Fimbriae, bacterial appendages that play a role in bacterial attachment, are the major antigens
against which agglutinating antibodies are directed. These agglutinating antibodies have historically been the primary
means of serotyping B. pertussis strains. Other virulence factors include tracheal cytotoxin, which causes respiratory
epithelial damage; adenylate cyclase toxin, which impairs host immune-cell function; dermonecrotic toxin, which may
contribute to respiratory mucosal damage; and lipooligosaccharide, which has properties similar to those of other gram-
negative bacterial endotoxins. 
Immunity against Bordetella: 
Both humoral and cell-mediated immunity are thought to be important in pertussis. Antibodies to pertussis toxin,
filamentous hemagglutinin, pertactin, and fimbriae are all protective in animal models. Pertussis agglutinins were
correlated with protection in early studies of whole-cell pertussis vaccines. Serologic correlates of protection conferred by
acellular pertussis vaccines have not been established, although antibody to pertactin, fimbriae, and (to a lesser degree)
pertussis toxin correlated best with protection in two efficacy trials. The duration of immunity after whole-cell pertussis
vaccination is short-lived, with little protection remaining after 10–12 years. After a three-dose infant primary series of
acellular pertussis vaccine, protection persists for at least 5–6 years; the duration of immunity after a four- or five-dose
schedule is not yet known. Although immunity after natural infection has been said to be lifelong, seroepidemiologic
evidence suggests that it may not be and that subsequent episodes of clinical pertussis are prevented by intermittent
subclinical infection. 
Clinical Manifestations 
21. Signature fracture refers to? 
A. Depressed skull fracture 
B. Suture displacement fracture 
C. Contrecoup injury 
D. Fracture at foramen magnum 
Ans: Depressed fracture (REPEAT) I don’t think the question needs an explanation 
22. Which among the following is most frequently seen in anti phospholipid antibody syndrome?(Pls CORRECT Q) 
A. Beta 2 microglobulin antibody 
B. Anti nuclear antibody 
C. Anti centromere antibody 
D. Anti beta 2 glycoprotein antibody 
Ans: D Anti beta 2 GP Ab 
The antiphospholipid antibody syndrome (APS) may be defined as the occurrence of arterial or venous thrombosis or
recurrent miscarriage in association with laboratory evidence of persistent antiphospholipid antibody. The antibody may
manifest itself as either a lupus anticoagulant detected by clotting tests or an anticardiolipin antibody (ACA) detected by
immunoassay. The syndrome may be associated with an autoimmune disorder, especially SLE (secondary), or may occur
independently of other autoimmune disorders (primary). A large proportion of antiphospholipid antibodies are actually
directed against complexes of phospholipid with protein, most notably beta-2-glycoprotein I. The ACA immunoassay
detects both these clinically significant antibodies and antibodies directed purely against cardiolipin and not beta-2-
glycoprotein I. The latter antibodies are transient, and are often associated with infection but not thrombosis. A positive
anti-b2 GPI assay is evidence in favor of APS, once persistence is demonstrated by repeating the assay after eight
weeks. 
23. What factor is responsible for deciding whether an antibody will remain membrane bound or get secreted? 
A. RNA splicing 
B. Class switching 
C. Differential RNA processing 
D. Allelic exclusion 
Ans: C Differential RNA Processing(REPEAT) 
24. Blood Chimerism is associated with? 
A. Monochorionic monoamniotic twins 
B. Monochorionic diamniotic twins 
C. Singleton pregnancy 
D. Vanishing twin 
Ans: B Monochorionic diamniotic twins (no doubt if these were the options) 
Explanation: Reference: 
“ Journal Ref: Blood Chimerism in a Dizygotic Dichorionic Pregnancy” 

“Blood chimerism in monochorionic twins conceived by induced ovulation: Case report” 


Journal says “ Blood chimerism “is more common in monochorionic dizygotic(MCDZ)twins and rare in dichorionic twins 
“Cases in which a monochorionic placenta occurs in a twin pregnancy,vascular anstomosis is well described and can lead
to blood chimerism as well as twin twin transfusion syndrome.In contrast interplacental vascular communications occur
only with very rare exceptions in a dichorionic placenta” 
25. In expectant management of placenta praevia, all are done except? 
A. Cervical encirclage 
B. Anti D 
C. Corticosteroids 
D. Blood transfusion 
Ans: a Cervical encirclage 
Reference: Williams Obstetrics 
The treatment of classical cervical incompetence is cerclage. The operation is performed to surgically reinforce 
the weak cervix by some type of purse-string suturing. Bleeding, uterine contractions, or ruptured membranes are
usually contraindications for cerclage. 
26. Which drug is not used during delivery in a woman with rheumatic heart disease ? 
A. Methylergometrine 
B. Carboprost 
C. Syntocin 
D. Misoprostol 
Ans: A Methylergometrine(REPEAT) 
27. Which is not an autoimmune disease? 
A. SLE 
B. Grave's disease 
C. Myasthenia gravis 
D. Sickle cell disease 
Ans: D Sickle Cell Disease(I don’t think an explanation is needed..Sickle cell ds is a hemoglobinopathy and not an
autoimmune ds) 
28. All are true regarding selective estrogen receptor downregulator (SERD),fulvestrant except? 
A. Used for breast cancer 
B. Is a selective oestrogen antagonist 
C. Is slower acting, safer, LESS effective than SERM 
D. Given as once a month im dose 
Ans: C Its slower acting and less efficacious than SERM 
Reference:This Question has been taken line to line from Goodman Gilman Pharmacology as u will see below: 
Fulvestrant 
Fulvestrant (FASLODEX) is the first FDA approved agent in the new class of estrogen-receptor downregulators, which
were hypothesized to have an improved safety profile, faster onset, and longer duration of action than the SERMs due to
their pure ER antagonist activity (Robertson, 2002). Fulvestrant was approved in 2002 for postmenopausal women with
hormone receptor-positive metastatic breast cancer that has progressed despite antiestrogen therapy. 
Mechanism of Action 
Fulvestrant is a steroidal antiestrogen that binds to the ER with an affinity more than 100 times that of tamoxifen, inhibits
its dimerization, and increases its degradation. 
Preclinical studies suggest that as a consequence of this ER "downregulation," ER-mediated transcription is abolished,
completely suppressing the expression of estrogen-dependent genes (Howell et al., 2004b). This difference in the activity
of fulvestrant likely explains why fulvestrant demonstrates efficacy against tamoxifen-resistant breast cancer. 
However, the hypothesis that fulvestrant provides more effective antiestrogen activity than tamoxifen was not confirmed
by a clinical trial comparing fulvestrant (250 mg intramuscularly monthly) with tamoxifen (20 mg orally daily) as first-line
therapy in metastatic breast cancer (Howell et al., 2004a). 
Absorption, Fate, and Excretion 
Maximum plasma concentrations are reached about 7 days after intramuscular administration of fulvestrant and are
maintained over a period of 1 month. The plasma half-life is approximately 40 days. Steady-state concentrations are
reached after 3 to 6 monthly injections. There is extensive and rapid distribution, predominantly to the extravascular
compartment. 
Various pathways, similar to those of steroid metabolism including oxidation, aromatic hydroxylation, and conjugation,
extensively metabolize fulvestrant. CYP3A4 appears to be the only CYP isoenzyme involved in the oxidation of fulvestrant.
Several preclinical and clinical studies have confirmed that fulvestrant is not subject to CYP3A4 interactions that might
affect the safety or efficacy of the drug. The putative metabolites possess no estrogenic activity and only the 17-keto
compound demonstrates a level of antiestrogenic activity about 4.5 times less than that of fulvestrant. The major route of
excretion is via the feces, with less than 1% being excreted in the urine (Robertson and Harrison, 2004). 
Therapeutic Uses 
Fulvestrant typically is administered as a 250-mg intramuscular injection at monthly intervals. It is used in
postmenopausal women as antiestrogen therapy of hormone receptor-positive metastatic breast cancer after progression
on first-line antiestrogen therapy such as tamoxifen (Strasser-Weippl and Goss, 2004). Fulvestrant is at least as effective
in this setting as the third-generation aromatase inhibitor anastrozole. 
Fulvestrant 250 mg (administered as a once-monthly 5-ml intramuscular injection) also has been compared with
tamoxifen 20 mg (orally once daily) in a trial of postmenopausal women with ER-positive and/or progesterone receptor
(PR)-positive or ER/PR-unknown metastatic breast cancer who had not previously received endocrine or chemotherapy.
There was no difference between fulvestrant and tamoxifen in time to disease progression in either the entire study
population or the subset of patients with ER- and/or PR-positive disease. Observed differences in other efficacy endpoints
favored tamoxifen, and fulvestrant equivalence was not demonstrated (Vergote and Robertson, 2004). The long time to
steady-state plasma levels for fulvestrant has brought into question the results of existing studies, and trials are in
progress to test the relative efficacy of giving an initial loading dose followed by regular monthly injections. 
Clinical Toxicity 
Fulvestrant generally is well tolerated with the most common adverse events being nausea, asthenia, pain, vasodilation
(hot flushes), and headache. Injection site reactions, seen in about 7% of patients, are reduced by giving the injection
slowly. In the study comparing anastrozole and fulvestrant, quality-of-life outcome measures were maintained over time
with no significant difference between the drugs . 
29. A farmer developed a swelling in the inguinal region which later ulcerated. What stain can be used to detect bipolar
stained organisms? 
A. Albert's stain 
B. Waysons stain 
C. Ziehl neelsen stain 
D. Nigrosin stain 
Ans: B Wayson stain(REPEAT) 
30. An 8 year old boy completed 8 out of 10 day course of cefaclor. Now he developed a generalized erythmatic rash
which is mildly pruritic and lymphadenopathy. Diagnosis is? 
A. Kawasaki disease 
B. Type 3 hypersensitivity 
C. Anaphylaxis 
D. Infectious mononucleosis 
Ans: B Type 3 Hypersensitivity(REPEAT) 
31. Rave drug is? 
A. Cannabis 
B. Cocaine 
C. Heroin 
D. Amphetamine 
Ans: D Amphetamine(MDMA) 
MDMA is also called Rave drug and ecstasy 
32. How to differentiate ASD from VSD in X-ray? 
A. Enlarged Left atrium 
B. Normal left atrium 
C. Pulmonary congestion 
D. Aortic shadow 
Ans: B Normal LA size 
LA remains normal in ASD despite volume overload since it can decompress through 2 outlets that is into RA and into
LV.So LA enlargement is not seen in ASD wheras it is seen in VSD 
33. Regarding an imbecile, all are true except? 
A. IQ is 50-60 B. Intellectual capacity equivalent to a child of 3-7 years of age 
C. Not able to take care of themselves 
D. Condition is congenital or acquired at an early age 
Ans: A IQ is 50-60 
Reference: Kaplan and saddock 

Q Range Classification 
70-80 Borderline deficiency 
50-69 Moron 
20-49 Imbecile 
below 20 Idiot 

34. Which is not true regarding diet modification recommended in high cardiovascular risk group? 
A. Cholesterol less then 100 mg/1000kcal/day 
B. Avoid alcohol 
C. Fat intake 10% of total calories 
D. Salt limitation to less than 5 gm 
Ans: C Fat intake <10% of total calories 
Reference: American Heart association 2006 guidelines 
These guidelines confirm the options 1/2/4...regarding option 3....it is the saturated fat content which should be less that
10%.the total fat intake should be less than 30% of the overall calorie intake. 
IF THE OPTION 3 WAS....Saturated Fat intake less than 10% then the answer shall become Avoid alcohol...as AHA
guidelines say upto 2-3 drink per day is fine.... 
35. Maintenance dose of which of the following drugs is used worldwide for opioid dependence? 
A. Naltrexone 
B. Methadone 
C. lmipramine 
D. Disulfiram 
Ans: B Methadone 
Reference: Kaplan and saddock 
Methadone is a synthetic narcotic (an opioid) that substitutes for heroin and can be taken orally. When given to addicts to
replace their usual substance of abuse, the drug suppresses withdrawal symptoms. A daily dosage of 20 to 80 mg suffices
to stabilize a patient, although daily doses of up to 120 mg have been used. The duration of action for methadone
exceeds 24 hours; thus, once-daily dosing is adequate. Methadone maintenance is continued until the patient can be
withdrawn from methadone, which itself causes dependence. An abstinence syndrome occurs with methadone
withdrawal, but patients are detoxified from methadone more easily than from heroin. Clonidine (0.1 to 0.3 mg three to
four times a day) is usually given during the detoxification period. 
Methadone maintenance has several advantages. First, it frees persons with opioid dependence from using injectable
heroin and, thus, reduces the chance of spreading HIV through contaminated needles. Second, methadone produces
minimal euphoria and rarely causes drowsiness or depression when taken for a long time. Third, methadone allows
patients to engage in gainful employment instead of criminal activity. The major disadvantage of methadone use is that
patients remain dependent on a narcotic. 
36. Best test/Gold standard test for assesing HCG function/action? 
A. Radioimmunoassay 
B. ELISA 
C. Latex test 
D. Bioassay 
Ans: D Bioassay(OPEN TO SCRUTINY......CHALLENGING REFERENCES INVITED) 
My explanation....Though Radioimmunoassay is more sensitive than Bioassay for quatifying an antigen ,Bioassay only can
asses an harmone both quantitatively and qualitatively....THE Question SPECIFICALLY asked about the BEST TEST OF
FUNCTION which can be assessed by BIOASSAY and not Radioimmunoassay. 
37. Vitamin K is involved in the posttranswerlational modification of? 
A. Glutamate 
B. Aspartate 
C. -- 
D. – 
Ans: A Glutamate(REPEAT) 
38. Spinal anaesthesia is given at which level? 
A. L1-2 
B. L2-4 
C. S1 
D. Midline thoracic segments 
Ans: B L2-4 
Reference: Miller anesthesia(Procedureconsult) 
• Spinal anesthetics have their effects at the spinal cord, which originates at the foramen magnum of the skull and the
brainstem and extends caudally to the conus medullaris. The distal termination varies from about the level of the 3rd
lumbar vertebrae (L3) in infants to the lower border of L1 in adults. The spinal cord is surrounded by three membranes
(from central to peripheral): the pia mater, arachnoid mater, and dura mater. It is believed that the arachnoid mater is
responsible for up to 90% of the resistance to drug migration in and out of the CSF. Inside the subarachnoid space are
the CSF, spinal nerves, a network of trabeculae between the two membranes, and blood vessels supplying the spinal
cord. Although the spinal cord ends at about L1 in adults, the subarachnoid space continues to about the second sacral
vertebrae (S2). 
• Posterior to the epidural space is the ligamentum flavum, which extends from the foramen magnum to the sacral
hiatus. Immediately posterior to the ligamentum flavum are the lamina and spinous processes of the vertebral bodies or
the interspinous ligaments. Posterior to these structures is the supraspinous ligament, which joins the vertebral spines. 
• Anatomic landmarks most important to performance of spinal anesthesia are the iliac crests, the midline of the back,
and the vertebral spinous processes. Palpation of the midline of the back identifies the spinous processes and vertebral
interspaces in most patients but may be difficult in obese patients. A line drawn between the upper borders of the iliac
crests across the midline of the back identifies the approximate level of L4 or the L4-L5 interspace. 
• Spinal anesthesia is usually performed at the level of the L3 or L4 vertebrae in the adult patient, because the spinal
needle is introduced below the level at which the spinal cord ends. 
39. a child presented with mild fever little breathlessness..... was treated and she improved over 4 days and later
deteriorated again with fever and more breathlessness. x ray showed hyperlucency. diagnosis? 
1.bronchiolitis obliterans 
2.alveolar proteinosois 
3.bronchitis 4. AsthmA 
Ans: a bronchiolitis obliterans(REPEAT) 
40. Which of the following passes through foramen magnum? 
A. Internal Carotid Artery 
B. Sympathetic chain 
C. Hypoglossal Nerve 
D. Vertebral Artery 
Ans: D Vertebral artery(REPEAT) 
41. McKeon's theory on reduced prevalence of TB? 
A. Increased awareness and knowledge 
B. Medical advancement answer 
C. Behavioural modification 
D. Social and environmental factor 
Ans: D Social and environmental factor 
Reference:” McKeown and the Idea That Social Conditions Are Fundamental Causes of Disease Bruce G. Link, PhD and Jo
C. Phelan, PhD “ 
THE MCKEOWN THESIS STATES that the enormous increase in population and dramatic improvements in health that
humans have experienced over the past 2 centuries owe more to changes in broad economic and social conditions than to
specific medical advances or public health initiatives.1 The thesis gives center stage to social conditions as root causes of
the health of populations. On the basis of new data and numerous revisitations, however, Colgrove2 tells us that the
thesis has been “overturned” and the theory “discredited.” Whither, then, the idea that social conditions require
prominence in any complete understanding of the health of populations? When we turn away from “the thesis,” do we
accept an “antithesis” asserting that the role of social conditions is insignificant? 
42. Which among the following is not a cause of fasting hypoglycemia? 
A. Glucagon excess 
B. Glucose 6 phospatase deficiency 
C. Ureamia 
D. Glycogen synthase deficiency 
Ans: A Glucagon excess 
Explanation: I am not quoting any book...But i will explain...if anyone has a contradictory reference pls post... 
Glucose 6 phosphatase def leads to inability to mobilize glucose and hence can cause 
Glycogen synthase def leads to glycogen deficiency and hence during fasting patient goes hypo 
In uremia liver metabolism is hampered and hence hypo is possible 
In glucagon excess there is HYPERglycemia and hence the answer. 
43. Mineralocorticoid receptor is not present in? 
A. Liver 
B. Colon 
C. Hippocampus 
D. Kidney 
Ans:Liver(REPEAT) 
44. Prolonged treatment with INH leads to deficiency of? 
A. Pyridoxine 
B. Thiamine 
C. Pantothenic acid 
D. Niacin 
Ans: A Pyridoxine 
Reference: Harrison 17th ed table of vitamins 
Vitamin B6 
Defeciency symptoms: Seborrhea, glossitis convulsions, neuropathy, depression, confusion, microcytic anemia RDA:<0.2
mg Factors contributing to deficiency:Alcoholism, isoniazid 

45. Which is the most reliable objective sign of identifying pulmonary plethora in chest X-ray? 
A. Diameter of the main pulmonay artery >16mm 
B. Diameter of the lt pulmonay artery >16mm 
C. Diameter of the decending Rt pulmonay artery >16mm 
D. Diameter of the decending Lt pulmonay artery >16mm 
Ans: C(REPEAT) 
46. Necrotizing lymphadenitis is seen in? 
A. Kimura disease 
B. Kikuchi disease 
C. Hodgkin disease 
D. Castelma 
Ans: B Kikuchi disease (a type of Modified REPEAT) 
Reference:Harrison 17th ed and eMEDICINE 
Kikuchi disease, also called histiocytic necrotizing lymphadenitis or Kikuchi-Fujimoto disease 
The most common clinical manifestation of Kikuchi disease is cervical lymphadenopathy 
Kimura disease is a chronic inflammatory disorder of unknown etiology that most commonly presents as painless,
unilateral cervical lymphadenopathy or subcutaneous masses in the head or neck region. The disorder received its current
name in 1948, when Kimura et al2 noted the vascular component and referred to it as an "unusual granulation combined
with hyperplastic changes in lymphoid tissue." 
Lymph-Node Tuberculosis (Tuberculous Lymphadenitis) from Harrison 17th ed: 
The most common presentation of extrapulmonary tuberculosis (>40% of cases in the United States in recent series),
lymph-node disease is particularly frequent among HIV-infected patients. In the United States, children and women
(particularly non-Caucasians) also seem to be especially susceptible. Once caused mainly by M. bovis, tuberculous
lymphadenitis is today due largely to M. tuberculosis. Lymph-node tuberculosis presents as painless swelling of the lymph
nodes, most commonly at posterior cervical and supraclavicular sites (a condition historically referred to as scrofula).
Lymph nodes are usually discrete and nontender in early disease but may be inflamed and have a fistulous tract draining
caseous material. Associated pulmonary disease is seen in >40% of cases. Systemic symptoms are usually limited to
HIV-infected patients. The diagnosis is established only by fine-needle aspiration or surgical biopsy. AFB are seen in up to
50% of cases, cultures are positive in 70–80%, and histologic examination shows granulomatous lesions. Among HIV-
infected patients, granulomas usually are not seen. Differential diagnosis includes a variety of infectious conditions,
neoplastic diseases such as lymphomas or metastatic carcinomas, and rare disorders like Kikuchi disease (necrotizing
histiocytic lymphadenitis), Kimura's disease, and Castleman's disease. 

47. NARP syndrome is seen in? 


A. Mitochondrial function disorder 
B. Glycogen storage disorder 
C. Lysosomal storage disorder 
D. Lipid storage disorder 
Ans: A Mitochondrial disorder(REPEAT) 
48. A 65 yrs old lady presented with a swollen and painful knee. On examination, she was found to have grade III
osteoarthritic changes. What is the “BEST COURSE OF ACTION” ? 
A. Conservative management 
B. Arthroscopic washing 
C. Partial knee replacement 
D. Total knee replacement 
Ans: D Total knee replacement 
THIS QUESTION IS NOT A REPEAT FROM MAY AIIMS.... 
This question has been adequately discussed in the institute classes and the answer is TKR....Whatever be the case they
are asking the BEST course of action and so no point in going for a conservative approach... 
49. Causes of primary amenorrhoea are all except? 
A. Rokintasky syndrome 
B. Kallaman syndrome 
C. Sheehan syndrome D.Turner syndrome 
Ans: C Sheehan syndrome(REPEAT) 
50. Integrase inhibitor approved for treatment of HIV is? 
A. Raltegravir 
B. Indinavir 
C. Lopinavir 
D. Elvitegravir 
Ans: A Raltegravir 
Reference: Pasting a net reference since the texts don’t have thedrug....anyway Ralteravir and maraviroc are given in
Sparsh Pharmac 
Raltegravir (MK-0518, brand name Isentress) is an antiretroviral drug produced by Merck & Co., used to treat HIV
infection.[1] It received approval by theU.S. Food and Drug Administration (FDA) in October 2007, the first of a new class
of HIV drugs, the integrase inhibitors, to receive such approval. 
51. Deoxygenated blood is not seen in 
a. Pulmonary artery 
b. Umbilical artery 
c. Umbilical vein 
d. Renal vein 
Ans: C Umbilical vein(No explanation needed) 
52. All of the following are pneumatic bones except? 
A. Frontal 
B. Ethmoid 
C. Mandible 
D. Maxilla 
Ans: C Mandible (No explanation needed) 
53. Which of the following is not a contraindication for pregnancy? 
A. WPW syndrome 
B. Pulmonary hypertension 
C. Eisenmenger syndrome 
D. Marfan syndrome with aortic root dilatation 
Ans: A WPW syndrome(REPEAT) 
54. Which of the following antihypertensive drugs is contraindicated in a patient on Lithium in order to prevent toxicity? 
A. Clonidine 
B. Beta blockers 
C. Calcium channel blockers 
D. Diuretics 
Ans: D Diuretics 
Reference: goodman and gilman 
Most of the renal tubular reabsorption of Li+ occurs in the proximal tubule. Nevertheless, Li+ retention can be increased
by any diuretic that leads to depletion of Na+, particularly the thiazides (see Chapter 28) (Siegel et al., 1998). Renal
excretion can be increased by administration of osmotic diuretics, acetazolamide, or aminophylline, although they are of
little help in the management of Li+ intoxication. Triamterene may increase excretion of Li+, suggesting that some
reabsorption of the ion may occur in the distal nephron. However, spironolactone does not increase the excretion of Li+.
Some nonsteroidal antiinflammatory agents can facilitate renal proximal tubular resorption of Li+ and thereby increase
concentrations in plasma to toxic levels 
55. Superior vena caval syndrome is most commonly caused by? 
A. Lymphoma 
B. Small cell lung ca 
C. Non small cell lung ca 
D. Secondary tumours 
Ans: B Small cell carcinoma(REPEAT) 
56. Which of the following is not an adverse effect of thalidomide? 
A. Diarrhoea 
B. Teratogenicity 
C. DVT 
D. Peripheral neuropathy 
Ans: A Diarrhea 
Reference: Goodman and Gilman 
The most common adverse effects reported in cancer patients are sedation and constipation (Franks et al., 2004), while
the most serious one is treatment-emergent peripheral sensory neuropathy, which occurs in 10% to 30% of patients with
MM or other malignancies in a dose- and time-dependent manner (Richardson et al., 2004). Thalidomide-related
neuropathy is an asymmetric, painful, peripheral paresthesia with sensory loss, commonly presenting with numbness of
toes and feet, muscle cramps, weakness, signs of pyramidal tract involvement, and carpal tunnel syndrome. The
incidence of peripheral neuropathy increases with higher cumulative doses of thalidomide, especially in elderly patients.
Although clinical improvement typically occurs upon prompt drug discontinuation, long-standing residual sensory loss can
occur. Particular caution should be applied in cancer patients with preexisting neuropathy (e.g., related to diabetes) or
prior exposure to drugs that can cause peripheral neuropathy (e.g., vinca alkaloids or bortezomib), especially since there
has been little progress in defining effective strategies to alleviate neuropathic symptoms. An increasing incidence of
thromboembolic events in thalidomide-treated patients has been reported, but mostly in the context of thalidomide
combinations with other drugs, including steroids and particularly anthracycline-based chemotherapy (Zangari et al.,
2001), and with very low incidence with single-agent thalidomide treatment. 
57. Blount’s disease is: 
A. Genu valgum 
B. Genu varum 
C. Genu recurvatum 
D. Menisceal injury 
Ans: B Genu varum 
Reference:eMEDICINE(don’t have the orthobook with me right now) 
Blount disease is an uncommon growth disorder characterized by disordered ossification of the medial aspect of the
proximal tibial physis, epiphysis, and metaphysis. This progressive deformity is manifested by varus angulation and
internal rotation of the tibia in the proximal metaphyseal region immediately below the knee 
58. A teenaged girl complains of pain in knee on climbing stairs and on getting up after sitting for a long time. What is
the probable diagnosis? 
A. Chondromalacia patellae 
B. Plica syndrome 
C. Bipartite patella 
D. Patello-femoral osteoarthritis 
Ans: A Chondromalacia patellae 
Reference: Current Orthopedic Diagnosis and management. 
Bilateral knee pain and stiffness in a teenage girl is suggestive of a few conditions of which Chondromalacia patellae is
one. 
59. Which of the following is not included in parenteral nutrition? 
A. Fat 
B. Carbohydrate 
C. Fibre 
D. Micronutrients 
Ans: C Fibre 
60. Sparrow marks are seen in? 
A. Gunshot injuries 
B. Stab injury of face 
C. Vitriolage 
D. Windshield glass injury 
Ans: D Winshield glass injury 
Reference: Forensic pathology by David.J.Williams 
“The occupants of the first row seats in a motor vehicle accident may demonstrate facial injuries due to contact with
windshield glass called sparrow’s foot.” 

61. Amphotericin B causes deficiency of? 


A. Na 
B. Ca 
C. K 
D. Mg 
Ans: C Potassium 
Reference: Goodman and Gilman 
Renal tubular acidosis and renal wasting of K+ and Mg2+ also may be seen during and for several weeks after therapy.
Supplemental K+ is required in one-third of patients on prolonged therapy. 
62. All are seen in injury to common peroneal nerve except? 
A. Loss of sensation over sole 
B. Foot drop 
C. Injury to neck of fibula 
D. Loss of dorsiflexion of toe 
Ans: A loss of sensation over the sole(No explanation needed) 
63. Cause of premature death in schizophrenia? 
A. Homicide 
B. Suicide 
C. Toxicity of antipsychotic drug 
D. Hospital acquired infection 
Ans:B Suicide 
Reference:Kaplan and saddock(A STAIGHT LINE TO LINE FROM THE TEXT) 
Suicide 

Suicide is the single leading cause of premature death among people with schizophrenia. Suicide attempts are made by
20 to 50 percent of the patients, with long-term rates of suicide estimated to be 10 to 13 percent. These numbers reflect
an approximately 20-fold increase over the suicide rate in the general population. Often, suicide in schizophrenia seems
to occur “out of the blue,” without prior warnings or expressions of verbal intent. The most important factor is the
presence of a major depressive episode. Epidemiological studies indicate that up to 80 percent of schizophrenia patients
may have a major depressive episode at some time in their lives. Some data suggest that those patients with the best
prognosis (few negative symptoms, preservation of capacity to experience affects, better abstract thinking) can
paradoxically also be at highest risk for suicide. The profile of the patient at greatest risk is a young man 
64. Epileptic potential is present in 
A. Desflurane 
B. Halothane 
C. Sevoflurane 
D. Ether 
Ans: Sevoflurane(No explanation needed) 
65. Which of the following anesthetic drugs is contraindicated in a patient with hypertension? 
A. Ketamine 
B. Propofol 
C. Etomidate 
D. Diazepam 
Ans: Ketamine(Ketamine is known to increase the BP...no explanation needed 
66. All of the following decrease bone resorption in osteoporosis except? 
A. Alendronate 
B. Etidronate 
C. Strontium 
D. Teriparatide 
Ans: D Teripartide 
Reference: Goodman and Gilman 
Parathyroid Hormone (PTH) 
Continuous administration of PTH or high circulating PTH levels achieved in primary hyperparathyroidism causes bone
demineralization and osteopenia. However, intermittent PTH administration promotes bone growth. Selye first described
the anabolic action of PTH some 80 years ago, but this observation was largely ignored and generally forgotten.
Beginning in the 1970s, studies focused on the anabolic action of PTH, culminating with FDA approval of synthetic human
34-amino-acid amino-terminal PTH fragment [hPTH(1–34), teriparatide] for use in treating severe osteoporosis (Hodsman
et al., 2005). Full-length PTH(1–84) is likely to be approved in the near future; its benefits over PTH(1–34) are unclear. 
Absorption, Fate, and Excretion 
Pharmacokinetics and systemic actions of teriparatide on mineral metabolism are the same as for PTH. Teriparatide is
administered by once-daily subcutaneous injection of 20 g into the thigh or abdomen. With this regimen, serum PTH
concentrations peak at 30 minutes after the injection and decline to undetectable concentrations within 3 hours, whereas
the serum calcium concentration peaks at 4 to 6 hours after administration. Based on aggregate data from different
dosing regimens, teriparatide bioavailability averages 95%. Teriparatide clearance averages 62 L/hour in women and 94
L/hour in men, which exceeds normal liver plasma flow, consistent with both hepatic and extrahepatic PTH removal. The
serum half-life of teriparatide is approximately 1 hour when administered subcutaneously versus 5 minutes when
administered intravenously. The longer half-life following subcutaneous administration reflects the time required for
absorption from the injection site. The elimination of PTH(1–34) and full-length PTH proceeds by nonspecific enzymatic
mechanisms in the liver, followed by renal excretion. 
Clinical Effects 
In postmenopausal women with osteoporosis, teriparatide increases BMD and reduces the risk of vertebral and
nonvertebral fractures. Several laboratories have examined the effects of intermittent PTH on BMD in patients with
osteoporosis. In these studies, teriparatide increased axial bone mineral, although initial reports of effects on cortical
bone were disappointing. Coadministration of hPTH(1–34) with estrogen or synthetic androgen led to impressive gains in
vertebral bone mass or trabecular bone. However, in some early studies there was only maintenance or even loss of
cortical bone. Vitamin D insufficiency in patients at baseline or pharmacokinetic differences involving bioavailability or
circulating half-life may have contributed to observed differences on cortical bone. The most comprehensive studies to
date established the value of daily hPTH(1–34) administration on total BMD, with significant elevations of BMD in lumbar
spine and femoral neck and with significant reductions of vertebral and nonvertebral fracture risk in osteoporotic women
(Neer et al., 2001) and men (Finkelstein et al., 2003). 
Candidates for teriparatide treatment include women who have a history of osteoporotic fracture, who have multiple risk
factors for fracture, or who failed or are intolerant of previous osteoporosis therapy. 
Adverse Effects 
In rats, teriparatide increased the incidence of bone tumors, including osteosarcoma (Vahle et al., 2004). The clinical
relevance of this finding is unclear, especially since patients with primary hyperparathyroidism have considerably higher
elevations of serum PTH without a greater incidence of osteosarcoma. Nonetheless, teriparatide should not be used in
patients who are at increased baseline risk for osteosarcoma (including those with Paget's disease of bone, unexplained
elevations of alkaline phosphatase, open epiphyses, or prior radiation therapy involving the skeleton). Full-length PTH(1–
84), which is in clinical trials, has not been associated with osteosarcomas. Other adverse effects have included
exacerbation of nephrolithiasis and elevation of serum uric acid levels. 
67.Ondoni cells and Haller cells are associated with the following structures respectively? 
A. Optic nerve and Orbital floor answer 
B. Optic nerve and Internal carotid artery 
C. Internal carotid artery and Optic nerve 
D. Orbital floor and Internal carotid artery 
Ans: Optic nerve and Orbital floor(REPEAT) 

68. Pain sensation from the ethmoid sinus is carried by : 


A. Frontal nerve 
B. Lacrimal nerve 
C. Nasociliary nerve 
D. Infraorbital nerve 
Ans: C Nasociliary nerve which divides into the ethmoidal branches to supply the ethmoidal sinus 

69. 28-Which among the following is not used to treat alcohol dependence? 
A. Flumazenil 
B. Acamprosate 
C. Naltrexone 
D. Disulfiram 
Ans: A Flumazenil(a MODIFIED REPEAT......last time this question came with Diazepam in the options) 

70. A 40yr old patient has a single kidney with an exophytic mass of 4 cm size at it’s lower pole. Which among the
following is the best course of action? 
A. Partial nephrectomy 
B. Radical nephrectomy with dialysis 
C. Radical nephrectomy with immediate renal transwerplant 
D. Observation 

Ans: A Partial nephrectomy since it is the current method of choice for tumors less than or equal to 4cm and at poles. 

71. 29-Which among the following is the most common fungal infection seen in immuno competent patients? 
A. Aspergillus 
B. Candida 
C. Cryptococcus 
D. Mucor 

Ans: A Aspergillus 
Reference: Harrison 17th ed 
The required size of the infecting inoculum is uncertain; however, only intense exposures (e.g., during construction work,
handling of moldy bark or hay, or composting) are sufficient to cause disease in healthy immunocompetent individuals. 
An increasing incidence of invasive aspergillosis in medical intensive care units suggests that, in patients who are not
immunocompromised, temporary abrogation of protective responses as a result of glucocorticoid use or a general anti-
inflammatory state is a significant risk factor. 

72. All are seen in Argyl Robertson pupil except? 


A. Near reflex normal 
B. Direct reflex absent 
C. Consensual reflex normal 
D. Vision normal 

Ans: C Consensual reflex is normal 


Argyll Robertson pupil 
This is caused by neurosyphilis and is characterised by the 
following: 
• Involvement is usually bilateral but asymmetrical 
• The pupils arc small and irregular. 
• Light-near dissociation. • The pupils are very difficult to dilate. 

73. 31-A 5 year old boy while having dinner suddenly becomes aphonic and is brought to the casulty for the complaint of
respiratory distress. What should be the appropriate management? 
A. Cricothyroidotomy 
B. Emergency tracheostomy 
C. Humidified oxygen 
D. Heimlich maneuver 

Ans: D Heimlich manoeveure(please read the explanation carefully) 

This is the AHA guidelines for management of any conscious choking patient....The question in the exam had a
conscious,aphonic patient in respiratory distress...Now how to manage.....Most of the discussions in the forum give other
answer feeling that Heimlich has to be performed at the site only...lets C the guidelines and find the answer... 

1. Recognise signs of choking 


2. Call for help 
3. Use Heimlich manoeveure until the patient recovers OR LOSES CONSCIOSNESS 
4. Place the victim in a supine position ..open the mouth and perform a finger sweep 
5. Open the airway and attempt to ventilate 
6. If unsuccessful give upto 5 heimlich manoeveures 
7. Repeat 5 and 6 
8. If expert has arrived he has to perform a laryngoscopy and try to remove the foreign body if not already removed 
9. FAILING ALL ATTEMPTS ONLY CRICOTHYOTOMY SHOULD BE PERFORMED. 

74.Which among the following is a branch from the trunk of brachial plexus? 
A. Subscapular nerve 
B. Long thoracic nerve 
C. Anterior thoracic nerve 
D. Nerve to subclavius 

Ans:D Nerve to subclavius.......If i am sure suprascapular nerve was not at all there in the options....so the answer is
clear.... 

75.Orthotolidine test is used for detecting: 


A. Chlorine 
B. Nitrites 
C. Nitrates 
D. Ammonia 
Ans: A Chlorine(No explanation needed) 
76. Which among the following is the most common tumour associated with neurofibromatosis in a child? 
A. Juvenile myelomonocytic leukemia 
B.Acute lymphoblastic leukemia 
C. Acute monocytic leukemia 
D. Acute myeloid leukemia 

Ans: Juvenile myelomonocytic leukemia 


Reference: Nelson 18th ed 
Juvenile Chronic Myelogenous Leukemia 
Juvenile chronic myelogenous leukemia (JCML), also known as juvenile myelomonocytic leukemia, is a clonal proliferation
of hematopoietic stem cells that typically affects children <2 yr of age. Patients with this disease do not have the
Philadelphia chromosome that is characteristic of CML. Patients with JCML present with rashes, lymphadenopathy, and
splenomegaly. Analysis of the peripheral blood often shows an elevated leukocyte count and also may show
thrombocytopenia and the presence of erythroblasts. The bone marrow shows a myelodysplastic pattern, with blasts
accounting for <30% of cells. No distinctive cytogenetic abnormalities are seen. JCML is rare, constituting <2% of all
cases of childhood leukemia. Therapeutic reports are largely anecdotal. Patients with neurofibromatosis type 1 have a
predilection for this type of leukemia. Stem cell transplantation offers the best opportunity for cure, but much less so
than for classic CML. 

77. Diaphragm develops from all except: 


A. Septum transwerversum 
B. Dorsal mesocardium 
C. Pleuroperitoneal membrane 
D. Cervical myotomes 

Ans: D Cervical myotomes 


Reference: Langman Embryology gives text confirming the options A/D ....now regarding options B and D is the
confusion....Langman says the nerve supply alone develops from the level of the cervical myotomes but not the
diaphragm....so i prefer the answer cervical myotomes> dorsal mesocardium 

78. Erythema nodosum is seen in all except: 


A. Pregnancy 
B. Tuberculosis 
C. SLE 
D. Chronic pancreatitis 

Ans: D Chronic pancreatitis 


Causes 
Currently, the most common cause of erythema nodosum is streptococcal infection in children and streptococcal infection
and sarcoidosis in adults.3 Numerous other causes have been reported.4 The causes reported most often in the literature
are as follows: 
• Bacterial infections: Streptococcal infections are one of the most common causes of erythema nodosum. Tuberculosis
was an important cause in the past, but it has decreased dramatically as a cause for erythema nodosum; however, it still
must be excluded, especially in developing countries.5,6 Yersinia enterocolitica is a gram-negative bacillus that causes
acute diarrhea and abdominal pain; it is a common cause of erythema nodosum in France and Finland.7,8,9 Mycoplasma
pneumoniae infection may cause erythema nodosum. Erythema nodosum leprosum clinically resembles erythema
nodosum, but the histologic picture is that of leukocytoclastic vasculitis. Lymphogranuloma venereum may cause
erythema nodosum. Salmonella infection may cause erythema nodosum. Campylobacter infection may cause erythema
nodosum. 
• Fungal infections: Coccidioidomycosis (San Joaquin Valley fever) is the most common cause of erythema nodosum in
the American Southwest. In approximately 4% of males and 10% of females, the primary fungal infection (which may be
asymptomatic or involve symptoms of upper respiratory infection) is followed by the development of erythema nodosum.
Lesions appear 3 days to 3 weeks after the end of the fever caused by the fungal infection. Histoplasmosis may cause
erythema nodosum. Blastomycosis may cause erythema nodosum. 
• Drugs: Sulfonamides and halide agents are an important cause of erythema nodosum. Drugs more recently described
to cause erythema nodosum include gold and sulfonylureas. Oral contraceptive pills are implicated in an increasing
number of reports. 
• Enteropathies: Ulcerative colitis and Crohn disease may trigger erythema nodosum. Erythema nodosum associated with
enteropathies correlates with flares of the disease. The mean duration of chronic ulcerative colitis before the onset of
erythema nodosum is 5 years, and erythema nodosum is controlled with adequate therapy of the colitis. Erythema
nodosum is the most frequent dermatologic symptom ininflammatory bowel diseases, and it is strongly associated with
Crohn disease.10 
• Hodgkin disease and lymphoma: Erythema nodosum associated with non-Hodgkin lymphoma may precede the
diagnosis of lymphoma by months. Reports of erythema nodosum preceding the onset of acute myelogenous leukemia
have been published.11 
• Sarcoidosis: The most common cutaneous manifestation of sarcoidosis is erythema nodosum. A characteristic form of
acute sarcoidosis involves the association of erythema nodosum, hilar lymphadenopathy, fever, arthritis, and uveitis,
which has been termed Löfgren syndrome. This presentation has a good prognosis, with complete resolution within
several months in most patients. HLA-DRB1*03 is associated with Löfgren syndrome. Most DRB1*03-positive patients
have resolution of their symptoms within 2 years; however, nearly half of DRB1*03-negative patients have an
unremitting course.12 
• Behçet disease (associated with erythema nodosum) 
• Pregnancy: Some patients develop erythema nodosum during pregnancy, most frequently during the second trimester.
Repeated episodes occur with subsequent pregnancies or with the use of oral contraceptives. 

79. Posterior relations of head of pancreas are all except? 


A. Common bile duct 
B. First part of duodenum 
C. Aorta 
D. Inferior vena cava 
Ans: B First part of Duodenum(NO Explanation needed) 
80. Multiple sebaceous cysts seen in: 
A. Gardner’s syndrome 
B. Turcot syndrome 
C. Muir Torre syndrome 
D. Cowden syndrome 
Ans:C Muir torre syndrome 
Reference:Sabiston 18th ed table in chapter of GI neoplasia 
Muir-Torre variant: sebaceous adenomas, keratoacanthomas, sebaceous epitheliomas, and basal cell epitheliomas 

81. Which among the following is the hallmark of acute inflammation? 


A. Vasoconstriction 
B. Stasis 
C. Vasodilation and increase in permeability 
D. Leukocyte margination 
Ans: C Vasodilatation and increased vascular permeability(REPEAT) 
82.True about epidural opioids are all except? 
A. Acts on dorsal horn substantia gelatinosa 
B. Can cause Itching 
C. Function of the intestines are not affected 
D. Can cause respiratory depression 

Ans: C Function of the intestine is not affected 

Reference: Pharmacology By KATZUNG(THE QUESTION HAS BEEN TAKEN LINE TO LINE FROM TEXT---READ BELOW) 

Because of their direct action on the superficial neurons of the spinal cord dorsal horn, opioids can also be used as
regional analgesics by administration into the epidural or subarachnoid spaces of the spinal column. A number of studies
have demonstrated that long-lasting analgesia with minimal adverse effects can be achieved by epidural administration of
3–5 mg of morphine, followed by slow infusion through a catheter placed in the epidural space. It was initially assumed
that the epidural application of opioids might selectively produce analgesia without impairment of motor, autonomic, or
sensory functions other than pain. However, respiratory depression can occur after the drug is injected into the epidural
space and may require reversal with naloxone. Effects such as pruritus and nausea and vomiting are common after
epidural and subarachnoid administration of opioids and may also be reversed with naloxone if necessary. Currently, the
epidural route is favored because adverse effects are less common. 

82. Which among the following does not cause hyperpyrexia? 


A. MAOI 
B. Alcohol 
C. atropine 
D. Amphetamine 
Ans: B Alcohol 
Reference:Harisson 17th ed 
Alcohol(ethanol) causes hypothermia and not hyperthermia 
83. All are true about neuronal tumors except? 
A. 90% are malignant 
B. 95% occur in the abdomen 
C. They secrete catecholamines 
D. They arise from sympathetic ganglions 

Ans: A 90% are malignant...as per the rule of 10...only 10% are malignant 
Reference:Harrison 17th ed 
Epidemiology 
Pheochromocytoma is estimated to occur in 2–8 out of 1 million persons per year, and about 0.1% of hypertensive
patients harbor a pheochromocytoma. Autopsy series reveal prevalence figures of 0.2%. The mean age at diagnosis is
about 40 years, although the tumors can occur from early childhood until late in life. The "rule of tens" for
pheochromocytomas states that about 10% are bilateral, 10% are extraadrenal, and 10% are malignant. However, these
percentages are higher in the inherited syndromes. 

84. Campylobacter jejuni false is-(contributions from Dr.confident) 

a.commonest cause of campylobacteriosis 


b.polutry source of infection 
c.humans are reservoir 
d.associated with GBS 

Answer:C humans are the only reservoir[REPEAT AIIMS09] 

85. Which of the following does not cause indoor air pollution? (dr.confident contribution) 
A. CO 
B. Nitrogen dioxide 
C. Radon 
D. Mercury vapor 

86. Most important and potential agent that can be used in bioterrorism: 
A. Plague 
B. Small pox 
C. TB 
D. Clostridium botulinum 

YET TO SOLVE 

88. Most common site of obstruction after TURP? 


A. Navicullar foss 
B. Bulb 
C. Prostatic membranous urethra 
D. Bladder neck 

YET TO SOLVE 

89. Intraoperative myocardial infarction is best diagnosed by: 


A. ECG 
B. Invasive arterial pressure 
C. Central venous pressure 
D. Transwer esophageal echo 

Ans:D TEE(REPEAT) Reference awaited 

90. Pseudoisomorphic phenomenon seen in 


A. Psoriasis 
B. Lichen planus 
C. Vitiligo 
D. Plane warts 

Ans:D Plane warts(reference awiated) 


91. Campylobacter jejuni false is- 

a.commonest cause of campylobacteriosis 


b.polutry source of infection 
c.humans are reservoir 
d.associated with GBS 

ans : C. as animals are reservoirs of Camplyobacter 

92. Some antigen was injected into a rabbit. What antibody will it produce initially? 
A. IgG 
B. IgM 
C. IgE 
D. IgD 
Ans: B IgM(IS this question any trick or just a question asking for the primary immune respose) 
91. A fire breaks out during laser vocal cord surgery. What is not to be done? 
A. Pouring sterile water into the oral cavity 
B. Removing endotracheal tube 
C. 100% oxygen after discontinuing anesthetic gases 
D. Treatment with steroid & antibiotic 
Ans: 
Reference: Could not get a Text reference....a journal paper on Management of Airway fire during Microlaryngeal surgery
provides some light 
Managing fire 

Remove source of fire and extinguish with water 


Stop ventilation,” turn off O2 “ 
Mask ventilate with air, “then 100% O2 once fire is extinguished” 
Laryngoscopy and rigid bronchoscopy to remove debris 
Lavage and fibreoptic bronchoscopy if indicated by airway injury 
Common pattern is worst injury at the surgical site and little distal injury 
If severe injury 
“Maintain ventilation” 
Consider low tracheostomy 
“IV corticosteroids may be helpful” 
CXR, ABG with co-oximetry for smoke inhalation assessment 

Option B is correct... 
Option A not sure... 
Option C is TRICKY......we should Discontinue the oxygen first that itself will extinguish the fire....once the fire is gone
then ventilate with 100% O2.... 
OptionD is correct 

Answer should be A/C depends on the exact wordings of the option C 

Current answer: C 

92. Some antigen was injected into a rabbit. What antibody will it produce initially? 
A. IgG 
B. IgM…………….answer 
C. IgE 
D. IgD 

93. Which insect among the following is not resistant to DDT? 


A. Musca domestica 
B. Phlebotomus…………..answer 
C. Culex 
D. Anopheles stephensi 

94. Which virus among the following is least likely to cross placenta? 
A. Rubella 
B. Herpes simplex…………………… 
C. HIV 
D. HBV 

95. About yaws all are true except: 


A. Caused by Treponema pertenue 
B. Transwermitted non-venerally 
C. Secondary yaws can involve bones 
D. Last stages involve heart and nerves 
Ans: D Late stages involve heart and nerves 
Reference:Harrison 17th ed 
Yaws 
Also known as pian, framboesia, or bouba, yaws is caused by T. pallidum subspecies pertenue and is characterized by the
development of one or several primary lesions ("mother yaw"), which is followed bythe appearance of multiple
disseminated skin lesions. All early skin lesions are infectious and may persist for many months; cutaneous relapses are
common during the first 5 years. Late manifestations, affecting 10% of untreated persons, are destructive and can
involve skin, bone, and joints. 
The infection is transmitted by direct contact with infectious lesions, often during play or group sleeping, and may be
enhanced by disruption of the skin by insect bites or abrasions. After an average of 3–4 weeks, the first lesion begins as a
papule—usually on an extremity—and then enlarges (particularly during moist warm weather) to become papillomatous
or "raspberry-like" (thus the name "framboesia") (Fig. 163-2). Regional lymphadenopathy develops, and the lesion
usually heals within 6 months; dissemination is thought to occur during the early weeks of infection. A generalized
secondary eruption, accompanied by generalized lymphadenopathy, appears either concurrent with or following the
primary lesion, may take several forms (macular, papular, or papillomatous), and may become secondarily infected with
other bacteria. Painful papillomatous lesions on the soles of the feet result in a painful crablike gait ("crab yaws"), and
periostitis may result in nocturnal bone pain and polydactylitis. 

Late yaws is manifested by gummas of the skin and long bone, hyperkeratoses of the palms and soles, osteitis and
periostitis, and hydrarthrosis. The late gummatous lesions are characteristically extensive. Destruction of the nose,
maxilla, palate, and pharynx is termed gangosa and is similar to the destructive lesions seen in leprosy and
leishmaniasis. 
96. Weight gain in pregnancy is related to all except? 
A. Ethnicity 
B. Smoking 
C. Socioeconomic status 
D. Pre conceptional weight 
Ans: B Smoking (REPEAT) 
97. A 3.8 kg baby of a diabetic mother developed seizures 16 hours after birth. Most probable cause is? 
A. Hypoglycemia 
B. Hypocalcemia 
C. Birth asphyxia 
D. Intra ventricular hemorrhage 

Ans: A Hypoglycemia(REPEAT) 
[snip]. Regarding PCOD, all are true except? 
A. High LH/FSH 
B. High DHEAS 
C. Very high prolactin 
D. Raised LH 
Ans: C Very high prolactin(REPEAT) 
99. Which anesthetic modality is to be avoided in sickle cell disease? 
A. General anesthesia 
B. Brachial plexus block 
C. IV regional anesthesia 
D. Spinal 
Ans: C IV regional anesthesia(REPEAT) 
100. Best marker for intrahepatic cholestasis of pregnancy is? 
A. Bile salts 
B. Bile acid 
C. Bilirubin 
D. Alkaline phosphatise 
Ans: B Bile acids(REPEAT) 
101. Which of the following is the most probable diagnosis in a patient with loss of central vision and a normal ERG with
no family history? 
A. Best's disease 
B. Stargardt's disease 
C. Retinitis pigmentosa 
D. Macular hole 
Ans:B Stargardt disease 
Clincal ophthalmology by Vaughan 
Both Eale’s and Stargrdt’s can manifest with loss of central vision and normal ERG....but stargerdt is most common and it
is autosomal recessive wheras Eales is AD and less common... 
102. Renal calculi associated with proteus infection is: 
A. Uric acid 
B. Triple phosphate 
C. Calcium oxlalate 
D. Xanthine 
Ans:B Triple phosphate(REPEAT) 
103. The primary action of NO in git is? 
A. Vasodilatation 
B. Vasoconstriction 
C. GI smooth muscle inhibition 
D. Secretomotor 
Ans:C GI smooth muscle inhibition(REPEAT) 
104. A 40 year old female underwent surgery. Post operatively she told the anaesthetist that she was aware of per-
operative events. Individual intraoperative awareness is evaluated by (to prevent such instances from occurring)? 
A. Pulse oximetry 
B. Colour doppler 
C. Bispectral index 
D. End tidal CO2 
Ans:C BIS 
Refernce: Quantitative EEG, event-related potentials and neurotherapy 
By Juri Kropotov 
Bispectral index is used to asses the depth of anesthesia.Its a good measure of brain activites in sleep wakefulness
cycle. 
105. All of the following helps in generating oxygen burst for killing bacteria within neurophils except? 
A. Superoxide dismutase 
B. Oxidase 
C. Peroxidase 
D. Glutathione peroxidise 
Ans: A Superoxide dismutase and Glutathione peroxidise(OPEN FOR DISCUSSION) 
Reference: Robbins pathos 8th ed 
A series of enzymes acts as free radical–scavenging systems and breaks down H2O2 and o2. .[21,][23] These enzymes
are lo-cated near the sites of generation of the oxidants and include the following: 
1. Catalase, present in peroxisomes, decomposes H2O2 (2H2O2 ➙ O2 + 2H2O). 
2. Superoxide dismutases (SOD) are found in many cell types and convert O2 to H2O2 (2O2. + 2H ➙ H2O2 + O2). This
group includes both manganese–SOD which is localized in mitochondria, and copper-zinc–SOD, which is found in the
cytosol. 
3. Glutathione peroxidase also protects against injury by catalyzing free radical breakdown (H2O2 + 2GSH ➙ GSSG
[glutathione homodimer] + 2H2O, or 2OH + 2GSH ➙ GSSG + 2H2O). The intracellular ratio of oxidized glutathione
(GSSG) to reduced glutathione (GSH) is a reflection of the oxidative state of the cell and is an important indicator of the
cell's ability to detoxify ROS. 

106. Most common cause of meningoencephalitis in children? 


A. HSV 
B. Enterovirus 
C. Mumps 
D. Listeria 
Ans:B Enterovirus(REPEAT) 

107. Principle orgenelle involved in the execution of apoptosis is? 


A. Nucleus 
B. Lysosome 
C. Mitochondria 
D. Endoplasmic reticulum 

Answer:C Mitochondria(REPEAT) 

108. Feature of obstructive azoospermia is? 


A. High FSH, high testosterone 
B. Low FSH, high testosterone 
C. High FSH, low testosterone 
D. Normal FSH, normal testosteron 

Ans:D NORMAL Testosterone and FSH(REPEAT) 

109. 'C' in C reactive protein stands for: 


A. Capsular polysaccharide in pneumococcus 
B. Concanavalin-a 
C. Calretinin 
D. Cellular 

Ans:A Capsular polysaccharide of pneumococcus(REPEAT) 

110. Buprenorphine is? 


A. Partial agonist at Mu Receptor 
B. Partial agonist at Kappa Receptor 
C. Full Agonist at Mu Receptor 
D. Antagonist at Kappa receptor 

Ans:A Partial agonist of mu receptor 


Reference: Goodman and Gilman 
Buprenorphine appears to be a partial receptor agonist. Depending on the dose, buprenorphine may cause symptoms of
abstinence in patients who have been receiving receptor agonists for several weeks. It antagonizes the respiratory
depression produced by anesthetic doses of fentanyl about as well as does naloxone without completely reversing opioid
pain relief (Boysen et al., 1988). Although respiratory depression has not been a major problem, it is not clear whether
there is a ceiling for this effect (as seen with nalbuphine and pentazocine). The respiratory depression and other effects
of buprenorphine can be prevented by prior administration of naloxone, but they are not readily reversed by high doses
of naloxone once the effects have been produced. This suggests that buprenorphine dissociates very slowly from opioid
receptors. The half-life for dissociation from the receptor is 166 minutes for buprenorphine, as opposed to 7 minutes for
fentanyl (Boas and Villiger, 1985). Therefore, plasma levels of buprenorphine may not parallel clinical effects.
Cardiovascular and other side effects (e.g., sedation, nausea, vomiting, dizziness, sweating, and headache) appear to be
similar to those of morphine-like opioids.

111.All are true about aprepitant except? 


A. Agonist at NK1 
B. Crosses blood brain barrier 
C. Ameliorates nausea and vomiting of chemotherapy 
D. Metabolized by CYP450 

Ans:A Agonist at NK1 Receptor 


Reference:Katzung Pharmacology 
Neurokinin 1 (NK1) receptor antagonists have antiemetic properties that are mediated through central blockade in the
area postrema. Aprepitant is a highly selective NK1 receptor antagonist that crosses the blood-brain barrier and occupies
brain NK1 receptors. It has no affinity for serotonin, dopamine, or corticosteroid receptors. 

112. Time of occurrence of secondary hemorrhage after tonsillectomy? 


A. 24 hrs 
B. 6 days 
C. 12 days 
D. 12 hrs 

Ans: 6 Days(No reference needed) 

113. A patient with head injury on examination revealed eye opening in response to pain, inappropriate words and pain
localisation. Calculate GCS? 
a. 10 
b. 8 
c. 12 
d. 14 

Ans: A 10(GCS—no referencs needed) 


114. A primigravida in 1st trimester had sputum positive for acid fast bacillus. What is the preferred treatment? 
A. Treatment deferred till 2nd trimester 
B. Category 1 DOTS 
C. Category 2 DOTS 
D. Category 3 DOTS 
Ans:B Category 1 DOTS(REPEAT) 

117. What will you give to treat hypothyroidism in a patient with ischemic heart disease? 
A. Low dose of levothyroxine 
B. Normal dose of levothyroxine 
C. Do not give levothyroxine 
D. Thyroid extract ??? 

118. Carrier state is not important in transmission of: 


A. Measles 
B. Typhoid 
C. Polio 
D. Diphtheria 

Ans:A Measlers(REPEAT) 

119. A schizophrenic patient started on haloperidol 2 days back, comes with complaints of torticollis and orofaciolingual
movements. What is the diagnosis? 
A. Acute dystonia 
B. Tardive dyskinesia 
C. Parkinsonism 
D. Akathisia 
Ans:A Acute Dystonia 
Reference:Kaplan and saddok 
Neuroleptic-Induced Acute Dystonia 
Diagnosis, Signs, and Symptoms 
Dystonias are brief or prolonged contractions of muscles that result in obviously abnormal movements or postures,
including oculogyric crises, tongue protrusion, trismus, torticollis, laryngeal–pharyngeal dystonias, and dystonic
postures of the limbs and trunk. Other dystonias include blepharospasm and glossopharyngeal dystonia; the latter results
in dysarthria, dysphagia, and even difficulty in breathing, which can cause cyanosis. Children are particularly likely to
evidence opisthotonos, scoliosis, lordosis, and writhing movements. Dystonia can be painful and frightening and often
results in noncompliance with future drug treatment regimens. 
Epidemiology 
The development of dystonic symptoms is characterized by their early onset during the course of treatment with
neuroleptics and their high incidence in men, in patients younger than age 30 years, and in patients given high dosages
of high-potency medications. 
Etiology 
Although it is most common with intramuscular doses of high-potency antipsychotics, dystonia can occur with any
antipsychotic. The mechanism of action is thought to be dopaminergic hyperactivity in the basal ganglia that occurs when
central nervous system (CNS) levels of the antipsychotic drug begin to fall between doses. 

120. All are true about parvovirus b19 except? 


A. <10 % spread by transplacental route 
B. Spread by respiratory route 
C. It is a DNA virus 
D. Affects erythroid progenitor cells 
Ans:A <10% spread by transplacental route(REPEAT)
121. Sterile pyuria is present in? 
A. Renal tuberculosis 
B. Chronic hydronephrosis 
C. Wilm's tumour 
D. Neuroblastoma 
Ans:A Sterile pyuria(REPEAT) 

122. A 35 year old female has proximal weakness of muscles, ptosis and easy fatiguability. The best test to diagnose her
condition is: 
A. Muscle biopsy 
B. CPK 
C. Edrophonium test 
D. EMG 
Ans:C Edrophonium test(REPEAT) 

123. Denominator in Maternal Mortality Rate? 


A. Total number of live births 
B. Total number of married women 
C. Total number of births 
D. Midyear population 
Ans:A Total live births(REPEAT) 

124. Psammoma bodies are seen in all except? 


A. Follicular carcinoma thyroid 
B. Papillary carcinoma thyroid 
C. Cystadenocarcinoma 
D. Meningioma 

Ans:A Follicular carcinoma thyroid(REPEAT) 

125. Visceral larva migranswer is seen in? 


A. Strongyloides 
B. Ancylostoma 
C. Toxocara canis 
D. Visceral leishmaniasis 

Ans:A Toxocara canis(no explanation needed) 


Reference:Jawetz microbiology 

126. Urethral crest is situated in: 


A. Prostatic urethra 
B. Membranous urethra 
C. Penile urethra 
D. Bulbar urethra 
Ans:A Prostatic urethra(no explanation needed) 

127. People were separated into relevant 5 sub groups. People were selected randomly from these sub groups. What type
of sampling was done? 
A. Simple random sampling 
B. Stratified Sampling 
C. Cluster sampling 
D. Systematic sampling 

Ans:Stratified sampling 

128. All of the following are true about erlotinib except? 


A. Tyrosine kinase inhibitor 
B. Food delays its absorption 
C. Rashes can occur 
D. Used in non small cell lung cancer when there is no response to other chemotherapeutic agents 

Ans:B Food delays absorption 


Reference:Goodman and Gilman 
Absorption, Fate, and Excretion 
Erlotinib is about 60% absorbed after oral administration and its bioavailability is substantially increased to almost 100%
by food. Peak plasma levels occur 4 hours after an oral dose. Following absorption, erlotinib is approximately 93%
protein-bound to albumin and alpha1-acid glycoprotein. Its half-life is ~36 hours. Erlotinib is metabolized primarily by
CYP3A4 and to a lesser extent by CYP1A2 and CYP1A1.
129. What will you give to treat hypothyroidism in a patient with ischemic heart disease? 
A. Low dose of levothyroxine 
B. Normal dose of levothyroxine 
C. Do not give levothyroxine 
D. Thyroid extract 

Ans:A Low dose of levothyroxine(OPEN FOR DISCUSSION) 

Reference---BRAUNWALD'sPRINCIPLES OF CARDIOLOGY 
Ans:- Fibroid
130. Arthropod transmitted disease not found in India? 
A. West nile fever 
B. Dengue 
C. Yellow fever 
D. – 
Ans:C Yellow fever(the question was pretty direct)

130.

1. Most common nerve injured in supracondylar fracture humerus? 


a. Median 
b. Radial 
c. Ulnar 
d. Anterior interosseus nerve 
Ans: D Anterior interosseus nerve 

I would not like to discuss much on this question since it has already been discussed in detail previously...The Mnemonic for the
order of nerves injured in Supracondylar fracture is AMRU 
That is: 
Ant . Interosseus Nerve> Median > Radial >Ulnar 

2. Earliest symptom of GERD in an infant is? 


A. Respiratory distress answer 
B. Upper GI bleed 
C. regurgitation 
D.obstruction 

Ans: C REGURGITATION.... 

Reference from Nelson Pediatrics 


Infantile reflux manifests more often with regurgitation (especially postprandially), signs of esophagitis (irritability, arching,
choking, gagging, feeding aversion), and resulting failure to thrive; symptoms resolve spontaneously in the majority by 12–24
mo. Older children, in contrast, may have regurgitation during the preschool years; complaints of abdominal and chest pain
supervene in later childhood and adolescence. Occasional children present with neck contortions (arching, turning of head)
designated Sandifer syndrome. The respiratory presentations are also age dependent: GERD in infants may manifest as
obstructive apnea or as stridor or lower airway disease in which reflux complicates primary airway disease such as
laryngomalacia or bronchopulmonary dysplasia. Otitis media, sinusitis, lymphoid hyperplasia, hoarseness, vocal cord nodules,
and laryngeal edema have all been associated with GERD. In contrast, airway manifestations in older children are more
frequently related to asthma or to otolaryngologic disease such as laryngitis or sinusitis. 
3. Basal metabolic rate is closely associated with? 
A. Lean body mass 
B. Body surface area 
C. Body mass index 
D. Body weight 
Ans: A Lean Body mass (REPEAT...shall not spend much time on repeats) 
4. A girl presented with recurrent occipital headache associated with ataxia and vertigo. Mother also has similar complaints. Most
probable diagnosis is? 
A. Vestibular neuronitis 
B. Basillar migraine 
C. TIA 
D. – 
Ans: b Basilar migraine(REPEAT) 

5. Drug of choice for central Diabetes Insipidus is? 


a. Desmopressin 
b. Leuperolide 
c. Thiazide diuretics 
d. – 
Ans: A Desmopressin 
Reference: Harrispn 17th ed 
The signs and symptoms of uncomplicated pituitary DI can be eliminated completely by treatment with desmopressin (DDAVP), a
synthetic analogue of AVP (Fig. 334-1). It acts selectively at V2 receptors to increase urine concentration and decrease urine flow
in a dose-dependent manner (Fig. 334-4). It is also more resistant to degradation than AVP and has a three- to fourfold longer
duration of action. Desmopressin (DDAVP) can be given by IV or SC injection, nasal inhalation, or oral tablet. 
6. A 32 year old mountaineer has a hematocrit of 70%. What is the possible explanation? 
A. Polycythemia with relative dehydration 
B. High altitude cerebral oedema 
C. High altitude pulmonary oedema 
D. Hemodilution 

Ans.A Polycythemia and relative dehydration 

Reference:Harrison 17th ed 


Polycythemia can be spurious (related to a decrease in plasma volume; Gaisbock's syndrome), primary, or secondary in origin.
The secondary causes are all associated with increases in EPO levels: either a physiologically adapted appropriate elevation
based on tissue hypoxia (lung disease, high altitude, CO poisoning, high-affinity hemoglobinopathy) or an abnormal
Mifepristone is used in? 
A. Molar pregnancy 
B. T

Most common nerve injured in supracondylar fracture humerus? 


a. Median………………answer 
b. Radial 
c. Ulnar 
d. Axilary 

2. Damage to Superior oblique nerve causes diplopia 


a. Horizontal and downward ????? 
b. Vertical and downward ? 
c. Horizontal and upward 
d. Vertical and upward 

3. Which of the following contraception method is contraindicated in women with epilepsy? 


a. Oral Contraceptive pill …………..answer 
b. IUCD 
c. Condom 
d. Mifepristone 

4. Deoxygenated blood is not seen in 


a. Pulmonary artery 
b. Umbilical artery 
c. Umbilical vein………answer 
d. Renal vein 

5. Which of the following is not supplied by the anterior division of mandibular nerve (V3) ? 
A. Temporalis 
B. Medial pterygoid………..answer 
C. Lateral pterygoid 
D. Masseter 

6. All of the following are pneumatic bones except? 


A. Frontal 
B. Ethmoid 
C. Mandible……….answer 
D. Maxilla 

7. Which of the following is not a contraindication for pregnancy? 


A. WPW syndrome………….answer 
B. Pulmonary hypertension 
C. Eisenmenger syndrome 
D. Marfan syndrome with aortic root dilatation 

8. Which of the following is a contraindication for medical treatment in gallstones? 


A. Radio opaque stones………. 
B. Radiolucent stones 
C. Normal functioning gall bladder 
D. Small stones answer…..(may cause obs of cbd) 

9. Low dose oral contraceptive pill contains? 


A. Levonorgestrel………… 
B. Norgestrel 
C. Desogestrel answer (feminon) 
D. Norethihisterone 

10. Which of the following antihypertensive drugs is contraindicated in a patient on Lithium in order to prevent toxicity? 
A. Clonidine 
B. Beta blockers 
C. Calcium channel blockers 
D. Diuretics………….answer 

11. Superior vena caval syndrome is most commonly caused by? 


A. Lymphoma 
B. Small cell lung ca………….answer 
C. Non small cell lung ca 
D. Secondary tumours 

12. Which of the following is not an adverse effect of thalidomide? 


A. Diarrhoea………….answer 
B. Teratogenicity 
C. DVT 
D. Peripheral neuropathy 

13. Blount’s disease is: 


A. Genu valgum 
B. Genu varum………..answer 
C. Genu recurvatum 
D. Menisceal injury 

14. A teenaged girl complains of pain in knee on climbing stairs and on getting up after sitting for a long time. What is
the probable diagnosis? 
A. Chondromalacia ?????? 
B. Plica syndrome 
C. Bipartite patella 
D. Patello-femoral osteoarthritis 

15. Which of the following is not included in parenteral nutrition? 


A. Fat 
B. Carbohydrate 
C. Fibre……………answer 
D. Micronutrients 

16. Sparrow marks are seen in? 


A. Gunshot injuries 
B. Stab injury of face 
C. Vitriolage 
D. Windshield glass injury…………..answer? 
17. Which organ obtained from a cadaver is not used for transwerplantation? 
A. Blood vessel ????? 
B. Lung 
C. Liver 
D. Bladder 

18. 17-Amphotericin B causes deficiency of? 


A. Na 
B. Ca 
C. K…………..answer 
D. Mg 

19. 18-All are seen in injury to common peroneal nerve except? 


A. Loss of sensation over sole……………….answer 
B. Foot drop 
C. Injury to neck of fibula 
D. Loss of dorsiflexion of toe 

20. 19-Cause of premature death in schizophrenia? 


A. Homicide 
B. Suicide……………answer 
C. Toxicity of antipsychotic drug 
D. Hospital acquired infection 

21. 20-Epileptic potential is present in 


A. Desflurane 
B. Halothane 
C. Sevoflurane…………….answer 
D. Ether 

22. 21-Which among the following is the best inotrope drug for use in right heart failure? 
A. Dobutamine 
B. digoxin 
C. Halothane 
D. Milrinone ????? 

23. 22-Which of the following anesthetic drugs is contraindicated in a patient with hypertension? 
A. Ketamine……………answer 
B. Propofol 
C. Etomidate 
D. Diazepam 

24. 23-Baby born at 33 weeks / 1.5 kg should be started on? 


A. Nil oral and IV fluids 
B. Oral nasogastric tube / alternate oral route………………?????? 
C. IV fluids and oral feeding ???? 
D. TPN 

25. 24-A patient has a single kidney with an exophytic mass of 4 cm size at it’s lower pole. Which among the following is
the best course of action? 
A. Partial nephrectomy…………….answer 
B. Radical nephrectomy with dialysis 
C. Radical nephrectomy with immediate renal transwerplant 
D. Observation 

26. 25-All of the following decrease bone resorption in osteoporosis except? 


A. Alendronate 
B. Etidronate 
C. Strontium 
D. Teriparatide………….answer 

27. 26-Ondoni cells and Haller cells are associated with the following structures respectively? 
A. Optic nerve and Orbital floor answer 
B. Optic nerve and Internal carotid artery 
C. Internal carotid artery and Optic nerve 
D. Orbital floor and Internal carotid artery 
28. 27-Pain sensation from the ethmoid sinus is carried by : 
A. Frontal nerve 
B. Lacrimal nerve 
C. Nasociliary nerve…………..answer 
D. Infraorbital nerve 

29. 28-Which among the following is not used to treat alcohol dependence? 
A. Flumazenil ………………..answer 
B. Acamprosate 
C. Naltrexone 
D. Disulfiram 

30. 29-Which among the following is the most common fungal infection seen in immuno competent patients? 
A. Aspergillus ????? 
B. Candida 
C. Cryptococcus 
D. Mucor 

31. 30-All are seen in Argyl Robertson pupil except? 


A. Near reflex normal 
B. Direct reflex absent 
C. Consensual reflex normal answer 
D. Vision normal 

32. 31-A 5 year old boy while having dinner suddenly becomes aphonic and is brought to the casulty for the complaint of
respiratory difficulty. What should be the appropriate management? 
A. Cricothyroidotomy 
B. Emergency tracheostomy 
C. Humidified oxygen ???? 
D. Heimlich maneuver………. 

33. 32-Which among the following is a branch from the trunk of brachial plexus? 
A. Suprascapular nerve…………..answer? 
B. Long thoracic nerve 
C. Anterior thoracic nerve 
D. Nerve to subclavius………………..answer? 

34. 33-Orthotolidine test is used for detecting: 


A. Chlorine…………answer 
B. Nitrites 
C. Nitrates 
D. Ammonia 

35. 34-Which among the following is the most common tumour associated with neurofibromatosis in a child? 
A. Juvenile myelomonocytic leukemia………….answer 
a. B. Acute lymphoblastic leukemia 
C. Acute monocytic leukemia 
D. Acute myeloid leukemia 

36. Diaphragm develops from all except: 


A. Septum transwerversum 
B. Dorsal mesocardium 
C. Pleuroperitoneal membrane 
D. Cervical myotomes answer 

37. ECG is poor at detecting ischaemia in areas supplied by? 


A. Left anterior descending 
B. Left circumflex 
C. Left coronary artery 
D. Right coronary artery answer 

38. A patient with history of discharge from right ear for past 1 year presented with severe ear ache. The discharge was
cultured and the organism was found to be gram positive cocci. The least likely cause is? 
A. Psuedomonas 
B. Streptococcus pneumoniae 
C. Staphylococcus 
D. Haemophilus influenza?………… 

39. Erythema nodosum is seen in all except: 


A. Pregnancy 
B. Tuberculosis 
C. SLE 
D. Chronic pancreatitis………..answer 

40. Posterior relations of head of pancreas are all except? 


A. Common bile duct 
B. First part of duodenum …………..answer 
C. Aorta 
D. Inferior vena cava 

41. Multiple sebaceous cysts seen in: 


A. Gardner’s syndrome 
B. Turcot syndrome 
C. Torre Maura syndrome………….answer 
D. Cowden syndrome 

42. Which among the following is the hallmark of acute inflammation? 


A. Vasoconstriction 
B. Stasis 
C. Vasodilation and increase in permeability………….answer 
D. Leukocyte margination 

43. A fire breaks out during laser vocal cord surgery. What is not to be done? 
A. Pouring sterile water …answer 
B. Removing endotracheal tube 
C. 100% oxygen after discontinuing anesthetic gases 
D. Treatment with steroid & antibiotic 

44. Which insect among the following is not resistant to DDT? 


A. Musca domestica 
B. Phlebotomus…………..answer 
C. Culex 
D. Anopheles stephensi 

45. Which virus among the following is least likely to cross placenta? 
A. Rubella 
B. Herpes simplex…………………… 
C. HIV ……….answer 
D. HBV 

46. Which among the following does not cause hyperpyrexia? 


A. MAOI 
B. Alcohol answer 
C. atropine 
D. Amphetamine 

47. Which is not seen in digoxin toxicity? 


A. Biventricular tachycardia 
B. Proxysmal atrial tachycardia 
C. Ventricular bigeminy 
D. Regularisation of AF ???? 
i. 
49. Which of the following does not cause indoor air pollution? 
A. CO 
B. Nitrogen dioxide answer 
C. Radon 
D. Mercury vapor 

51. All are true about neuronal tumors except? 


A. 90% are malignant ……answer 
B. 95% occur in the abdomen 
C. They secrete catecholamines 
D. They arise from sympathetic ganglions 

52. Most important and potential agent that can be used in bioterrorism: 
A. Plague 
B. Small pox ,,,,answer 
C. TB 
D. Clostridium botulinum 

53. True about epidural opioids are all except? 


A. Acts on dorsal horn cell 
B. Itching 
C. Nausea & vomiting answer 
D. Respiratory depression 

54. Most common site of obstruction after TURP? 


A. Navicullar foss 
B. Bulb 
C. Prostatic membranous urethra????? 
D. Bladder neck 

55. Intraoperative myocardial infarction is best diagnosed by: 


A. ECG ?? 
B. Invasive arterial pressure ? 
C. Central venous pressure 
D. Transwer esophageal echo 

56. Pseudoisomorphic phenomenon seen in 


A. Psoriasis ………answer 
B. Lichen planus 
C. Vitiligo 
D. Plane warts 

57. Some antigen was injected into a rabbit. What antibody will it produce initially? 
A. IgG 
B. IgM…………….answer 
C. IgE 
D. IgD 

58. Regarding PCOD, all are true except? 


A. High LH/FSH 
B. High DHEAS 
C. Very high prolactin…………………….answer 
D. Raised LH 

59. Which anesthetic modality is to be avoided in sickle cell disease? 


A. General anesthesia 
B. Brachial plexus block 
C. Local arterial injection answer 
D. Spinal 

60. About yaws all are true except: 


A. Caused by Treponema pertenue 
B. Transwermitted non-venerally 
C. Secondary yaws can involve bones 
D. Last stages involve heart and nerves answer 

61. Rise in end tidal CO2 during thyroid surgery can be due to all except: 
A. Anaphylaxis answer 
B. Malignant hyperthermia 
C. Thyroid storm 
D. Neuroleptic malignant syndrome 

62. Weight gain in pregnancy is related to all except? 


A. Ethnicity 
B. Smoking …………………………..answer 
C. Socioeconomic status 
D. Pre conceptional weight 

63. A 3.8 kg baby of a diabetic mother developed seizures 16 hours after birth. Most probable cause is? 
A. Hypoglycemia answer 
B. Hypocalcemia………………………… 
C. Birth asphyxia 
D. Intra ventricular hemorrhage 

64. Best marker for intrahepatic cholestasis of pregnancy is? 


A. Bile salts 
B. Bile acid……………………………..answer 
C. Bilirubin 
D. Alkaline phosphatase 

66. A 6 year old child presents with pain in hip in femoral triangle region. X-ray does not reveal any abnormality. What is
the next step? 
A. USG 
B. MRI 
C. Aspiration answer 
D. Traction 

68. All are true except: 


A. Human anatomical waste is disposed in a yellow bag 
B. Red bag contents can be a source of contamination 
C. Black bag is used for incineration ash 
D. Blue bag contents are always disposed in secure landfill answer 

69. A patient presented to the casuality with bluish pigmentation of conjunctiva, mucous membranes, nails and
tachycardia after ingestion of a poison. What is the poison: 
A. Mercury 
B. Arsenic answer 
C. Lead 
D. Copper 

70. The distant metastasis of bone can be best detected by: 


A. MRI 
B. Bone scan answer 
C. CT 
D. Intravenous venogram 

71. Which of the following is the most probable diagnosis in a patient with loss of central vision and a normal
retinogram? 
A. Best's disease 
B. Stargardt's disease ………… answer 
C. Retinitis pigmentosa 
D. Macular hole 

72. Methods of producing monoclonal antibodies are all except? 


A. Attaching inactive viral particle on cell membrane 
B. Adding ethylene glycol …answer 
C. Applying a small electric current 
D. Reducing the viscosity of the membrane 

73. Dose of radiation required for development of haematological syndrome is? 


A. 2.5-5 Gy 
B. 10 Gy 
C. 100 Gy 
D. 200 cGy …………answer 

74. The primary action of NO in git is? 


A. Vasodilatation 
B. Vasoconstriction 
C. GI smooth muscle relaxation…………………..answer 
D. Secretomotor 
76. Renal calculi associated with proteus infection is: 
A. Uric acid 
B. Triple phosphate…………………answer 
C. Calcium oxlalate 
D. Xanthine 

77. Thiamine deficiency causes decreased energy production because? 


A. It is required for the process of transweramination 
B. It is a co-factor in oxidative reduction 
C. It is a co-enzyme for transwerketolase in pentose phosphate pathway 
D. It is a co-enzyme for pyruvate dehydrogenase & alpha ketoglutarate dehydrogenase……..answer 

78. Regarding anterior choroidal artery syndrome, all are true except? 
A. Hemipareisis 
B. Hemisensory loss, 
C. Involvement of anterior limb of internal capsule…………….answer 
D. Homonymous hemianopia 

79. A 40 year old female underwent surgery. Post operatively she told the anaesthetist that she was aware of per-
operative events. Individual intraoperative awareness is evaluated by (to prevent such instances from occurring)? 
A. Pulse oximetry 
B. Colour doppler 
C. Bispectral imaging answer 
D. End tidal CO2 

80. Which of the following helps in generating oxygen burst for killing bacteria within neurophils ? 
A. Superoxide dismutase 
B. Oxidase …………answer 
C. Peroxidase 
D. Glutathione reductase 

81. Most common cause of meningoencephalitis in children? 


A. HSV 
B. Enterovirus ………….answer 
C. Mumps 
D. Listeria 

82. A 15 day old baby came with history of seizures. Blood tests revealed Ca 5mg/dl, PO4 9mg/dl, PTH 30pg/ml (n=10-
60). What is the most probable diagnosis? 
A. Pseudohypoparathyroidism ………answer 
B. Vitamin D deficiency 
C. Hyperparathyroidism 
4. HIE 

83. A graph of Normal blood sugar level curve and Diabetic blood sugar level curve was shown. An area was seen
overlapping towards the normal gycemic curve. A point at 120 mg/dl was shown too. Question : What does that area
represent? 
A. True positive 
B. False positive ………………….answer 
C. True negative 
D. False negative 

84. Which among the following not used in diagnosis of insulinoma? 


A. Fasting glucose test 
B. Xylulose test………………………answer 
3. C peptide levels 
4. Insulin / glucose ratio 

85. All are true about Nesidioblastosis except ? 


A. Hypoglycemic episodes are seen 
B. Occurs in adults more than children ………………answer 
C. Histopathology shows hyperplasia of islet cells 
D. Diazoxide is used for treatment 

86. Which among the following is preferred in a patient with decreased renal functio to avoid contrast nephropathy? 
A. N acetylcysteine 
B. Fenoldopam answer…………… 
C. Low osmolar contrast 
D. Mannitol 

87. A young lady presents with fever, dysuria and pain abdomen. Uncomplicated acute cystitis was diagnosed. Which
among the following is not true? 
A. Nitrate test positive 
B. E.coli count was < 10^3 ……………answer 
C. 1 pus cell per 7 fields 
D. 1 bacilli per field 

88. A patient presented with an abdominal injury with peritonitis and shock. Airway, breathing and IV fluids for circulation
were taken care of. What is the next step of management? 
A. Take the patient for laparotomy under GA answer 
B. Take the patient go for a laparoscopy 
C. Insert an abdominal drain under LA and take up for surge... 

89. Mother to baby transwermission of HIV can be minimised by all except? 


A. Zidovudine 
B. Vitamin A 
C. Vaginal delivery …………..answer 
D. Avoidance of breast feeding 

91. A investigator finds out that 5 independent factors influence the occurrence of a disease. Comprision of multiple
factors responsible for a disease can be assessed by? 
A. ANOVA 
B multiple linear regression……..answer? 
C. Chi square 
D. multiple logistic regression 

92. A primigravida at 37 weeks of gestation with loss of engagement. Cervix 1cm dilated. 10 uterine contractions per
hour. What is management? 
A. Sedate the patient and wait ……..answer 
B. LSCS 
C. Amniotomy 
D. Induction with membrane rupture 

93. Gold standard test for diagnosis of laryngopharyngeal reflux? 


A. 24 hr double probe pH monitoring ……………….answer 
B. Flexible endoscopy 
C. Barium swallow 
D. Laryngoscopy 

94. Acoustic neuroma involves 


A. Superior vestibular division of 8th cranial nerve………………. answer 
B. Auditory part of 8th cranial nerve 
C. 7th cranial nerve 
d. Inferior vestibular division of 8th cranial nerve… 

95. A patient had running nose and pain over medical aspect of eye. He later developed chemosis,protosis and diplopia
on abduction of right eye with congestion of optic disc. What is the probable diagnosis? 
A. Acute ethmoidal sinusitis 
B. Orbital cellulitis 
C. Cavernous sinus thrombosis………….answer 
D. Orbital apex syndrome 

96. An anesthesia resident was giving spinal anaesthesia when the patient had sudden aphonia and loss of consciousness.
What could have happened? 
A. Total spinal ………………answer 
B. Partial spinal 
C. Vaso vagal attack 
D. Intra vessel injection 

97. Urea cycle occurs in: 


A. Liver……………..answer 
B. G.I.T. 
C. Spleen 
D. Kidney 

[[snip]]. Main blood supply of neck of femur? 


A. Lateral circumflex femoral 
B. Medial circumflex femoral 
C. Profunda femoris answer?? 
D. Popliteal artery 

99. Not a part of national screening program? 


A. Diabetes mellitus……………. 
B. Dental caries ?????? 
C. Refractive error... 
D. Carcinoma cervix 

100. Definitive airway is all except? 


A. Nasotracheal tube 
B. Orotracheal tube 
C. LMA …………………….answer 
D. Cricothyroidotomy 

101. Right isomerism is? 


A. Asplenia …………………..answer 
B. Two spleens 
C. One spleen 
D. Polysplenia 

103. Least common cause of ambiguous genitalia in a female child? 


A. Placental steroid sulfatase deficiency …….answer 
B. Fetal aromatase deficiency 
C. WT-4 mutation 
D. CAH 

104. Otogenic abcess is caused by all except? 


A. Pseudomonas 
B. Streptococcus 
C. Hemophilus inflenzae 
D. Staphylococcus 

105. True regarding leptospirosis is? 


A. Rats are the only reservoirs answer 
B. Fluroquinolones are the DOC 
C. Person to person transwermission 
D. Hepatorenal syndrome occurs in 50% cases 

106. Principle orgenelle involved in the execution of apoptosis is? 


A. Nucleus 
B. Lysosome 
C. Mitochondria………………answer 
D. Endoplasmic reticulum 

108. Feature of obstructive azoospermia is? 


A. High FSH, high testosterone 
B. Low FSH, high testosterone 
C. High FSH, low testosterone 
D. Normal FSH, normal testosterone…………….answer 

109. 'C' in C reactive protein stands for: 


A. Capsular polysaccharide in pneumococcus…………….answer 
B. Concanavalin-a 
C. Calretinin 
D. Cellular 

111. Buprenorphine is? 


A. Partial agonist at Mu Receptor ………………..answer 
B. Partial agonist at Kappa Receptor 
C. Full Agonist at Mu Receptor 
D. Antagonist at Kappa receptor 

112. All are true about aprepitant except? 


A. Agonist at NK1 answer 
B. Crosses blood brain barrier 
C. Ameliorates nausea and vomiting of chemotherapy 
D. Metabolized by CYP450 

113. Time of occurrence of secondary hemorrhage after tonsillectomy? 


A. 24 hrs 
B. 6 days answer 
C. 12 days 
D. 12 hrs 

114. Which of the following is true? 


A. Acetylcholinesterase inhibition by malathion can be reversed by increasing the level of atropine 
B. Sulphonilamide inhibits folate reductase irrevesibly answer 
C. Flouoroacetate competetively inhibits aconitase 
D. Ethanol inhibits aldehyde dehydrogenase when used in methanol poi... 

115. A patient with head injury on examination revealed eye opening in response to pain, inappropriate words and pain
localisation. Calculate GCS? 
a. 10 ………………answer 
b. 8 
c. 12 
d. 14 

116. A primigravida in 1st trimester had sputum positive for acid fast bacillus. What is the preferred treatment? 
A. Treatment deferred till 2nd trimester 
B. Category 1 DOTS…………..answer 
C. Category 2 DOTS 
D. Category 3 DOTS 

117. What will you give to treat hypothyroidism in a patient with ischemic heart disease? 
A. Low dose of levothyroxine 
B. Normal dose of levothyroxine 
C. Do not give levothyroxine 
D. Thyroid extract ??? 

118. Carrier state is not important in transwermission of: 


A. Measles…………answer 
B. Typhoid 
C. Polio 
D. Diphtheria 

119. A schizophrenic patient started on haloperidol 2 days back, comes with complaints of torticollis and orofaciolingual
movements. What is the diagnosis? 
A. Acute dystonia …….answer 
B. Tardive dyskinesia 
C. Parkinsonism 
D. Akathisia 

120. A 55 year old man presents with history of 5 episodes of hematuria each lasting for about 4-5 days in the past 5
years. What will be the best investigation to arrive at a diagnosis? 
A. Urine examination and microscopy…………. 
B. X-ray KUB 
C. Abdominal USG………. 
D. DTPA scan answer 

121. Mifepristone is used in? 


A. Molar pregnancy 
B. Threatened abortion 
C. Fibroid ………….answer?? 
D. Ectopic pregnancy… 
122. All are true about parvovirus b19 except? 
A. <10 % spread by transwerplacental route answer 
B. Spread by respiratory route 
C. It is a DNA virus 
D. Affects erythroid progenitor cells 

123. Sterile pyuria is present in? 


A. Renal tuberculosis…………………….answer 
B. Chronic hydronephrosis 
C. Wilm's tumour 
D. Neuroblastoma 

124. A 35 year old female has proximal weakness of muscles, ptosis and easy fatiguability. The best test to diagnose her
condition is: 
A. Muscle biopsy 
B. CPK 
C. Edrophonium test ………………answer 
D. EMG 

125. Denominator in Maternal Mortality Rate? 


A. Total number of live births……………answer 
B. Total number of married women 
C. Total number of births 
D. Midyear population 

126. Psammoma bodies are seen in all except? 


A. Follicular carcinoma thyroid……………..answer 
B. Papillary carcinoma thyroid 
C. Cystadenocarcinoma 
D. Meningioma 

127. Visceral larva migranswer is seen in? 


A. Strongyloides 
B. Ancylostoma 
C. Toxocara canis…………….answer 
D. Visceral leishmaniasis 

128. Urethral crest is situated in: 


A. Prostatic urethra …………answer 
B. Membranous urethra 
C. Penile urethra 
D. Bulbar urethra 

129. People were separated into 5 sub groups. People were selected randomly from these sub groups. What type of
sampling was done? 
A. Simple random sampling 
B. Stratified Sampling answer 
C. Cluster sampling 
D. Systematic sampling 

130. All of the following are true about erlotinib except? 


A. Tyrosine kinase inhibitor 
B. Food delays its absorption ……………answer 
C. Rashes can occur 
D. Used in non small cell lung cancer when there is no response to other chemotherapeutic agents 

131. Compliance is decreased in all except? 


A. Pulmonary congestion 
B. COPD answer 
C. Decreased surfactant 
D. Pulmonary fibrosis 

132. A 70yr old presents with intemittent jerks of recent origin, EEG showing bilateral periodic spikes. What is the most
probable diagnosis? 
A. Hepes simplex encephalitis 
B. Lewy body dementia 
C. Alzheimer's 
D. CJD………………answer 

133. Not a disorder of protein misfolding? 


A. Alzheimer's disease 
B. Tuberculosis…………….answer? 
C. Cystic fibrosis 
D. CJD 

134. Pulmonary toxicity is seen with? 


A. Bleomycin ……………answer 
B. Cisplatin 
C. Methotrexate 
D. Actinomycin D 

135. 18 year old male presents with hemetemesis, melena and splenomegaly. What is the most probable diagnosis? 
A. NCPF ………………….answer 
B. Cirrhosis 
C. Malaria with DIC 
D. Extra hepatic portal venous obstruction 

136. Following are true about carbohydrate antigen except? 


A. Memory ………………….answer 
B. Poly clonal response 
C. Poor immunogenicity 
D. T cell independent immunity 

137. Ideal age for surgery in unilateral undescended testis is? 


A. 6 months 
B. 12 months………………..answer 
C. 24 months 
D. 36 months 

138. Not a predisposing factor for atherosclerotic plaque formation? 


A. ApoE 
B. Alpha 2-macroglobulin……………answer 
C. Oxidised LDL 
D. Increased homocystiene 

139. What is the type of joint seen in the growth plate? 


A. Fibrous 
B. Primary cartilagenous …………….answer 
C. Secondary cartilagenous 
D. Plane joint 

140. Which of the following is a vector used to increase the yield of protein produced in recombinant protein synthesis? 
A. Promoter induced 
B. Genes for protease inhibitors 
C. Transwerlation initiation ???? 
D. Transwerlation and transwercription termination 

141. A poison which is illuminous, transwerlucent and waxy? 


A. Iodine 
B. Ammonium bromide 
C. Cobra venom 
D. Yellow phosphorous………………answer 

142. A 5 year old child presented with ballooning of perpuce while micturition. Perpuce adhesions were present. What is
the best treatment for him? 
A. Adhesiolysis and dilatation 
B. Circumcision ……………answer 
C. Dorsal slit 
D. Conservative 
143. A schizophrenic patient started on haloperidol 2 days back, comes with complaints of torticollis and orofaciolingual
movements. What is the diagnosis? 
A. Acute dystonia..........answer 
B. Tardive dyskinesia 
C. Parkinsonism 
D. Akathisia 

144 Earliest to be diagnosed by USG is? 


A. Anencephaly ………………….answer 
B. Prosencephaly 
C. Meningocele 
D. Spina bifida 

145. A 45 year old lady presented with DUB & USG finding of 8mm thick endometrium. What is the next step? 
A. Endometrial histopathology………………….answer 
B. Hysterectomy 
C. OCP 
D. Follow up 

146. Fallopian tube immotility is seen in? 


A. Churg strauss syndrome 
B. Kartagener's syndrome……………………..answer 
C. Noonan syndrome 
D. Turner syndrome 

147. First structure to be fixed after amputation is? 


A. Bone fixing………………….answer 
B. Arterial repair 
C. Venous repair 
D. Nerve repair 

148 Pearson's skewness coefficient is? 


A. (Mean-median)/SD 
B. Median-mean/SD 
C. SD/mean-median 
D. SD/median-mean 

149. Poor prognostic factor for ALL is? 


A. Hyperdiploidy 
B. t(9;22) t(4;11) ……..answer 
C. 2-8 yrs of age 
D. TLC < 50000 

150. Most potent activator of T cells? 


A. B cells 
B. Follicular dendritic cells 
C. Mature dendritic cells …………..answer 
D. Macrophages 
151. During TURP, surgeon takes care to dissect above the verumontenum so as to prevent injury to? 
A. External urethral sphincter 
B. Urethral crest answer 
C. Prostatic utricle 
D. trigone of bladder 

152. A neonate delivered at 38 weeks of gestation, birth weight of 2.2kg develops intolerance to feeds on 2nd day.
Physical examination reveals no abnormalities. Sepsis screen in negative. What is the next step in management? 
A. Wait and watch answer 
B. Do a 2nd sepsis screen 
C. give prophylactic antibiotics 
D. 
153. True about platelet function defect? 
A. Normal platelet count with prolonged bleeding time …………answer 
B. Thrombocytosis with prolonged bleeding time 
C. 
D. 

154. The acid base status of a patient is as follows : pH - 7.45, pCO2 - 30 mm of Hg, pO2 - 105 mm of Hg. Patient has
partially compensated? 
A. Metabolic acidosis answer 
B. Metabolic alkalosis 
C. Respiratory acidosis 
D. Respiratory alkalosis 

156. A patient presents with signs of pneumonia. The bacterium obtained from sputum grows on sheep agar. What test is
used to identify the type of organism? 
A. Bile solubility………………answer 
B. Bacitracin sensitivity 
C. Coagulase test 
D. 

157. A female presents with sings of meningitis. CSF shows gram positive bacilli. It is most probably? 
A. Listeria …………answer 
B. Haemophilus influenzae 
C. Pneumococcus 
D. 

158. All are used in the treatment of hot flushes except? 


A. Tamoxifene ………………..answer 
B. Venlafaxine 
C. 
D. 

159. A patient comes with history of fever and cough unresponsive to antibiotics.It was partially Acid fast. X-ray shows
consolidation. Bronchioalveolar lavage shows gram positive branching filaments. What is the diagnosis? 
A. Actinomycosis 
B. Nocardiosis 
C. Aspergillus answer 
D. 

160. About Human Development Index, all are true except? 


A. Life expectancy at birth 
B. Life expectancy at 1 year of age ……answer 
C. Education 
D. GDP 

161. Which among the following is not a component of hypogastric sheath? 


A. Broad ligament …………..answer 
B. Transwerverse cervical ligament 
C. lateral ligament 
D. Ligament of bladder 

162. All are seen in the floor of 3rd ventricle except? 


A. Infundibulum 
B. Oculomotor nerve 
C. Mammillary body 
D. Optic chiasm ????? 

163. Late onset endophthalmitis after lens implantation is caused by? 


A. Staphylococcus epidermidis 
B. Pseudomonas 
C. Streptococcus pyogenes 
D. Propionibacterium acnes 

164. All are true about blood coagulation except? 


A. Factor 10 in a part of both intrinsic and extrinsic pathway ….answer 
B. Extrinsic pathway is activated by contact with plasma and negatively charged proteins 
C. Calcium is very important for coagulation 
D. Intrinsic pathway can be activated in vitro 

165. Best treatment option for genuine stress incontinence? 


A. Burch colposuspension 
B. Kelly’s procedure answer 
C. Sling operation 
D. Tension free vaginal taping 

166. All of the following are done in management of shoulder dystocia except? 
A. Fundal pressure ……………….answer 
B. Suprapubic pressure 
C. McRoberts maneuver 
D. Woods maneuver 

167. Pentalogy of fallot has which one of following extra entities: 


A. ASD …………………..answer 
B. VSD 
C. RVH 
D. Pulmonary stenosis 

168. Aortic knuckle shadow on chest X ray, PA view is obliterated by consolidation of which portion of lung? 
A. Upper lingula 
B. Lower lingula ????? 
C. Apex of lower lobe 
D. Posterior part of upper lobe 

171. Best test for HCG action? 


A. Radioimmunoassay……………….answer 
B. ELISA 
C. Latex test 
D. Bioassay 

172. Lines of blaschko are: 


A. Lymphatics 
B. Blood vessel 
C. Nerves 
D. Lines of development…………….answer 

173. A man connected to a body plethysmograph exhales against a closed glottis. What will be the finding? 
A. The pressure in both the lungs and the box increases 
B. The pressure in both the lungs and the box decreases 
C. The pressure in the lungs decreases, but that in the box increases 
D. The pressure in the lungs increases, but that in the box decrease………………….answer 

174. A patient presents with fever and abdominal pain. Clinical examination reveals hepatomegaly extending 4 finger
breadths below the costal margin. USG reveals a 4cm*5cm*4cm hypodense lesion 1cm deep to liver surface. Tests for
hydatid disease were -ve. Best course of action is? 
A. Hepatectomy 
B. Multiple aspirations and me.dication answer 
C only medication 
d.. 

175. Main site of water absorption is: 


A. Jejunum………………..answer 
B. Colon 
C. Ileum 
D. Stomach 

176. All are true about delirium tremens except? 


A. Visual hallucinations 
B. Coarse tremors 
C. 3 Ocular palsy answer 
D. Unconsciousness 

177. All of the following are neuronal tumours except? 


A. Gangliocytoma 
B. Ganglioglioma 
C. Neuroblastoma 
D. Ependymoma…………..answer?? 

178. All are true about meglitinides except? 


A. Decreases post parandial hyperglycemia 
B. Hypoglycemia less common than sulfonylureas 
C. It decreases insulin resistance 
D. It acts by releasing insulin nas 

179. Pasteurised milk is most commonly tested by: 


A. Phosphatase test……………….answer 
B. Coliform test 
C. Catalase test 
D. Oxidase Test 

180. All of the following are affected in low radial nerve palsy except? 
A. Extensor carpi radialis longus answer 
B. Extensor carpi radialis brevis 
C. Finger extensors 
D. Sensation on dorsum of hand 

181. Clue Cells are seen in : 


A. Bacterial vaginosis ……………….answer 
B. Vaginal candidiasis 
C. Chlamydial vaginosis 
D. Trichomoniasis 

182. Which complement component is involved in both classical and alternate pathway? 
A. C1 
B. C2 
C. C3…………………..answer 
D. C4 

183. Which of the following are not associated with menstrual cycle? 
A. Hormonal changes 
B. Vaginal cytology changes 
C. Estrus profile……………….answer 
D. Endometrial changes 

184. HbH is seen in? 


A. Deletion of 3 alpha gene 
B. Deletion of all 4 alpha genes answer 
C. Deletion of 3 beta genes 
D. Deletion of all 4 beta genes 

185. Which among the following is a cardioprotective fatty acid? 


A. Palmitic acid 
B. Stearic acid 
C. Oleic acid 
D. Omega-3 fatty acids………………..answer 

186. All are true about xanthogranulomatous inflammation except? 


A. Presence of foamy macrophages 
B. Presence of tuberculous infection………………..answer 
C. Multinucleated giant cell 
D. Presence of yellow Nodules 

187. Site not affected in posterior cerebral artery infarct is? 


A. Midbrain 
B. Pons… 
C. Thalamus 
D. ant Cortex ???? 

188. Pregnant mother at 35 weeks of gestation. What drug can you not give her for treatment of SLE? 
A. Prednisolone 
B. Methotrexate…………………..answer 
C. Sulfsalazine 
D. Hydroxychloroquine 

189. Common carotid artery is palpated at? 


A. Upper border of cricoid cartilage 
B. Upper border of thyroid cartilage……………..answer 
C. Hyoid bone 
D. Cricothyroid membrane 

190. A 50 yr lady has history of sprained ankle 2 months back followed by recovery. She now complains of severe pain in
that ankle with inability to flex that foot. Physician notes edema and shiny skin in local examination. What is the probable
diagnosis: 
A. Fibromyalgia 
B. Complex regional pain syndrome 1 
C. Complex regional pain syndrome 2 answer 
D. Peripheral neuropathy 

191. Tolerance in opioids develops to all except? 


A. Miosis ……………………..answer 
B. Analgesia 
C. Euphoria 
D. Nausea and vomiting 

192. All are actions of muscarinic antagonist except? 


A. Decreases gastric secretion 
B. Prolongs a-v conduction ????? 
C. Decreases respiratory secretions 
D. Contraction of radial muscles of iris 

193. Which among the following is an early sign of magnesium toxicity? 


A. Depression of deep tendon reflexes answer 
B. Respiratory depression 
C. Cardiac arrest 
D. Decreased urine output 

194. Drugs used in prophylaxis of migraine are all except? 


A. Propranolol 
B. Flunarizine 
C. Topiramate 
D. Levetiracetam………………answer 

195. Auto-Rikshaw ran over a child’s thigh, there is a mark of the tyre tracks, it is an 
A. Contact bruise 
B. Patterned bruise 
C. Imprint abrasion……………….answer 
D. Ectopic bruise 

196. Anaesthetic agent with vasoconstrictor is contraindicated in? 


A. Finger block ……………..answer 
B. Spinal block 
C. Epidural block 
D. Regional anaesthesia 

197. Integrase inhibitor approved for treatment of HIV is? 


A. Raltegrase……………….answer 
B. Indinavir 
C. Lopinavir 
D. Elvitegravir 
198. A 65 yrs old lady presented with a swollen and painful knee. On examination, she was found to have grade III
osteoarthritic changes. What is the best course of action? 
A. Conservative management answer 
B. Arthroscopic washing 
C. Partial knee replacement 
D. Total knee replacement 

199. Causes of primary amenorrhoea are all except? 


A. Rokintasky syndrome 
B. Kallaman syndrome 
C. Sheehan syndrome……………answer 
4. Turner syndrome 

200. NARP syndrome is seen in? 


A. Mitochondrial function disorder……………………..answer 
B. Glycogen storage disorder 
C. Lysosomal storage disorder 
D. Lipid storage disorder 

201. Necrotizing lymphadenitis is seen in? 


A. Kimura disease 
B. Kikuchi disease ………………………answer 
C. Hodgkin disease 
D. Castelman disease 

202. Two plants are grown. One in some fluorescent pigment containing media. Other in fire fly luciferase containing
media. Which plant will glow in the dark? 
A. Both plants will glow 
B. Neither will glow 
C. First one will glow 
D. Second one will glow…………………..answer 

203. CT scan is least accurate for diagnosis of: 


A. 1 cm size aneurysm in an artery 
B. 1 cm size lymph node in thyroid carcinoma answer 
C. 1 cm size mass in tail of pancreas 
D. 1 cm size gall stone 

204. Which is the most reliable objective sign of identifying pulmonary plethora in chest X-ray? 
A. Diameter of the main pulmonay artery >16mm 
B. Diameter of the lt pulmonay artery >16mm 
C. Diameter of the decending Rt pulmonay artery >16mm…………………answer 
D. Diameter of the decending Lt pulmonay artery >16mm 

205. Prolonged treatment with INH leads to deficiency of? 


A. Pyridoxine …………………….answer 
B. Thiamine 
C. Pantothenic acid 
D. Niacin 

206. Mineralocorticoid receptor is not present in? 


A. Liver…………………….answer 
B. Colon 
C. Hippocampus 
D. Kidney 

207. A man presents with a maculopapular rash. He gives a history of previous painless rash. Infection is due to? 
A. Treponema pallidum …………..answer 
B. Chlamydia 
C. Calymmatobacterium granulomatis 
D. Haemophilus ducreyi 

208. In L5 root involvement, which among the following is not affected? 


A. Thigh adduction answer 
B. Knee flexion 
C. Knee extension 
D. Toe extension 

209. Which among the following is not a cause of fasting hypoglycemia? 


A. Glucagon excess 
B. Glucose 6 phospatase deficiency 
C. Ureamia 
D. Glycogen synthase deficiency answer 

210. McKeon's theory on reduced prevalence of TB? 


A. Increased awareness and knowledge 
B. Medical advancement answer 
C. Behavioural modification 
D. Social and environmental factor 

211. Which of the following passes through foramen magnum? 


A. Internal Carotid Artery 
B. Sympathetic chain 
C. Hypoglossal Nerve 
D. Vertebral Artery……………………answer 

212. A child presents with abdominal pain only during passage of stools. No other symptoms like vomiting or blood in
stools. There are no signs of intestinal obstruction. Most probable diagnosis is? 
A. Rectal polyp 
B. Intusseception answer 
C. Meckels diverticulum 
D. NEC 

213. Spinal anaesthesia is given at which level? 


A. L1-2 
B. L2-4 ……………………answer 
C. Midline of thorax 
D. Below L5 (caudal) 

215. a child presented with mild fever little breathlessness..... was treated and she improved over 4 days and later
deteriorated again with fever and more breathlessness. x ray showed hyperlucency. diagnosis? 
1.bronchiolitis obliteranswer 
2.alveolar proteinosois 
3.bronchitis 

216. coarctation of aorta mc asso with 


1.bicuspid aoric valve………………answer 
2.pda 
3. 
4. 

217. Vitamin K is involved in the posttranswerlational modification of? 


A. Glutamate…………………….answer 
B. Aspartate 
C. 
D. 

218. Muscular component of dorsal aorta develops from? 


A. Axial mesoderm 
B. Paraxial mesoderm answer 
C. Intermediate mesoderm 
D. Lateral plate mesoderm 

219. Maintenance dose of which of the following drugs is used worldwide for opioid dependence? 
A. Naltrexone 
B. Methadone………………..answer 
C. lmma 
D. Disulfiram 

220. Which is not true regarding diet modification recommended in high cardiovascular risk group? 
A. Cholesterol less then 100 mg/1000kcal/day 
B. Avoid alcohol 
C. Fat intake 10% of total calories ???? 
D. Salt limitation to less than 5 gm ???? 

221. Regarding an imbecile, all are true except? 


A. IQ is 50-60 ………………………………answer 
B. Intellectual capacity equivalent to a child of 3-7 years of age 
C. Not able to take care of themselves 
D. Condition is congenital or acquired at an early age 

222. All are true about ranalozine except? 


A. Causes hypotension 
B. It is recommended as first line treatment for angina 
C. Improves glycemic control 
D. anti angina answer 

223. How to differentiate ASD from VSD in X-ray? 


A. Enlarged Left atrium…………………….. 
B. Normal left atrium…………………….answer 
C. Pulmonary congestion 
D. Aortic shadow 

224. Rave drug is? 


A. Cannabis 
B. Cocaine 
C. Heroin 
D. Ecstasy………………….answer 

225. Mr X is a chronic smoker. His family insists on quitting smoking. He is thinking about quitting, but is reluctant to do
so because he is worried that on quitting he will become irritable. This is? 
A. Precontemplation and preparation……………………….answer 
B. Contemplation and extent of sickness susceptability 
C. Contemplation and cost factors 
D. Precontemplation and cost factors 

226. A farmer developed a swelling in the inguinal region which later ulcerated. What stain can be used to detect bipolar
stained organisms? 
A. Albert's stain 
B. Waysons stain…………………….answer 
C. Ziehl neelsen stain 
D. Nigrosin stain 

227. An 8 year old boy completed 8 out of 10 day course of cefaclor. Now he developed a generalized erythmatic rash
which is mildly pruritic and lymphadenopathy. Diagnosis is? 
A. Kawasaki disease 
B. Type 3 hypersensitivity………………………….answer 
C. Anaphylaxis 
D. Infectious mononucleosis 

228. All are true about world health report 2008 except? 
A. Social reforms 
B. Leadership ???? 
C. Polices 
D. Economic reforms 

229. Small air way has laminar flow because? 


A. Reynold number more than 2000 
B. Diameter is very small……………………..answer 
C. Density very high / velocity very high 
D. Total cross sectional area low 

230. Which is not an autoimmune disease? 


A. SLE 
B. Grave's disease 
C. Myasthenia gravis 
D. Sickle cell disease……………………answer 
231. All are true regarding selective estrogen receptor downregulator (SERD), fulvestrant except? 
A. Used for breast cancer 
B. Is a selective oestrogen antagonist 
C. Is slower acting, safer, more effective than SERM 
D. Given as once a month dose 

232. In pseudohyperparathyroidism, true is? 


A. Gain of function mutation 
B. Decreased conversion of GTP to GMP 
C. Decreased inositol tri phosphate production 
D. Decreased response to increase cAMP ??? 

233. Vectors don't transwermit infection by? 


A. Ingestion 
B. Regurgitation ????? 
C. Rubbing of feces 
D. Contamination with body fluids 

234. All are factors responsible for resurgence of malaria except? 


A. Drug resistance 
B. Use of bed nets…………………….answer 
C. Vector resistance 
D. Mutation in parasite 

235. Administration of which of the following drug needs alkalization of urine? 


A. Cytosine arabinoside 
B. Methotrexate………………………answer 
C. Cisplatin 
D. Ifosfamide 

236. Which drug is not used to control bleeding during delivery in a woman with heart disease ? 
A. Methylergometrine…………………..answer 
B. Carboprost 
C. Syntocin 
D. Misoprostol 

237. In expectant management of placenta praevia, all are done except? 


A. Cervical encirclage answer 
B. Anti D 
C. Corticosteroids 
D. Blood transwerfusion 

238. A 5 yr old boy presented with leukocoria in right eye ball, while other eye had 2-3 small lesions in the periphery.
What will be the ideal management for this patient? 
A. Enucleation of both eyes 
B. Enucleation of right eye & conservative management for the other eye 
C. Enucleation for right eye and radiotherapy for the other eye………………….answer?? 
D. 6 cycles of chemotherapy 

239. Chimerism is associated with? 


A. Monochorionic monoamniotic twins 
B. Monochorionic diamniotic twins 
C. Singleton pregnancy 
D. Vanishing twin answer 

240. A tennis player gets hit by a ball in the face following which he has complaints of decreased vision. which of the
following tells that injury is due to injury by ball. 
1. Optic neuritis 
B. Pars planitis ? 
C. Vitreous base detachment 
D. Equatorial edema ? 

241. What factor is responsible for deciding whether an antibody will remain membrane bound or get secreted? 
A. RNA splicing 
B. Class switching 
C. Differential RNA regulation……………answer 
D. Allelic exclusion 

242. Signature fracture refers to? 


A. Depressed skull fracture………………..answer 
B. Suture displacement fracture 
C. Contrecoup injury 
D. Fracture at foramen magnum 

243. Which among the following is seen in anti phospholipid antibody syndrome? 
A. Beta 2 microglobulin antibody 
B. Anti nuclear antibody 
C. Anti centromere antibody 
D. Anti glycoprotein antibody…………………….answer 

244. All are true regarding phagocytosis by protozoa except? 


A. Amoeba n other unicellular org make their living out of it 
B. Phagocytose particles of < 5 microns size answer 
C. Phagocytose particles of > 5 microns size 
D. Digestion occurs within phagolysosomes 

245. Child brought to casualty with reports of violent shaking by parents. Most likely injury is? 
A. Long bone fracture answer 
B. Ruptured spleen 
C. Subdural hematoma 
D. Skull bone fracture 

246. Gun powder on clothing can be visualized by? 


A. Magnifying lens 
B. UV rays 
C. Infrared rays 
D. dye answer 

247. Capsular antibody protection is seen in all except? 


A. Neisseria meningitidis 
B. Pneumococcus 
C. Bordetella pertussis answer 
D. Haemophilus influenza 

248. Which of the following is not an evidence based treatment for menorrhagia? 
A. Ethamsylate……………answer 
B.OCP 
C.Tamoxefene 
D. 

249. A lady who presented with hematuria on evaluation was found to have stage 2 transweritional cell carcinoma of
bladder. Which of the following is true? 
A. 70% chance of requiring cystectomy in 5 yrs………………….answer 
B. Cystoscopic fulguration will have to be done 
C. A 10 year history of beedi smoking is not a risk factor 
D. There is no chemotherapy available 

250. A sewer worker presented with fever. Lab findings revealed renal failure with increased BUN and serum creatinine.
What is the most appropriate drug to give him? 
A. Cotrimoxazole 
B. Erythromycin …answer 
C. Ciprofloxacin 
D.Benzyl penicillin 

251. True regarding chlamydia trachomatis is? 


A. Culture of endocervical discharge is used for isolation of organism 
B. Patient using OCP's are carriers answer 
C. 
D. 
253. A female patient presented with depressed mood, loss of appetite and no interest in surroundings. There is
associated insomnia. The onset of depression was preceeded by a history of business loss. What is the line of
management? 
A. No treatment is necessary as it is due to business loss 
B. SSRI is the best choice………………answer 
C. Start SSRI treatment based on side effect profile 
D. Combination therapy of 2 anti depressant drugs 

254. Antidepressant drug that can be used in nocturnal eneuresis? 


A. Imipramine…………..answer 
B. Fluoxamine 
C. 
D. 

255. Difference between follicular carcinoma and follicular adenoma is? 


A. Vascular invasion…………..answer 
B. 
C. 
D. 

256. If Gs alfa subunit gain of function mutation, there is? 


A. Decreased cAMP ?? 
B. Decreased IP3 
C. Increased gtpas activity ?? 
257 Cavitation is seen in? 
A. Mycolplasma pneumonia 
B. Tuberculous pneumonia 
C. Streptococcal pneumonia 
D.Staphylococcus pneumonia…………….answer 

258. Punnet Square is used for? 


A. Genotype of offspring ………………answer? 
B. Satistical analysis 
C. 
D. 

260. All are non deleberate measures for control of mosquito except? 
A. Use of alkaline soap water in factory 
B. Use of larvicidal agents 
C. Community participation 
D. use of bed nets for mosquito 

261. All of the following are true regarding diabetes mellitus except? 
A. Insulin is not used in type 2 diabetes………….answer 
B. Sliding scale regimen is used in hospitals 
C. 
D. 

262. A surgeon removed the part of liver to the left of the falciform ligament. Which segments have been removed? 
A. 1 & 4a 
B. 2 & 3…………….answer 
C. 1 & 4b 
D. 

263. A 6 week old male infant was brought in a state of dehydration and shock. Examination revealed hyper pigmentation
over the body with normal external genitalia. Blood tests revealed hypoglycemia, Na - 124 mEq/L and K - 7 mEq/L. What
is the probable diagnosis ? 
A. Congenital adrenal hyperplasia answer 
B. Adrenal haemorrhage and shock 
C. Acute gastroenteritis with dehydration 
D. 
264. A child presented with mild fever and breathlessness. He was treated and his condition improved over 4 days. Later
his condition deteriorated with increase in fever and breathlessness. X-ray showed hyperlucency. What is the probable
diagnosis? 
A. Bronchiolitis obliteranswer………………….answer 
B. Alveolar proteinosis 
C. Bronchitis 
D. 

265. All are true regarding serotonin syndrome except? 


A. It is not idiosyncratic and unpredictable 
B. Dantrolene is used for treatment……………answer 
C. 
D. 

266. False regarding Japanese encephalitis is: 


A. Epidemic is 2-3 cases in a village 
B. Mosquito bite is always associated with disease answer 
C. 70 in infants 
D. 

267. Which of the following is given to treat thrombocytopenia secondary to myelosuppresive therapy? 
A. Filgrastim 
B. Oprelvekin answer 
C. Erythropoietin 
D. 

268. True regarding ranula? 


A. It is also known as epulis 
B. It is a cystic swelling in the floor of mouth……………answer 
C. It is a type of thyroglossal cyst 
D. It is a type of mucus retention cyst 

269. A patient who was given primaquin develops hemolysis. Diagnosis is 
A. Glucose 6 phosphate dehydrogenase deficiency……………..answer 
B. Glucose 6 phosphatase deficiency 
C. 
D. 

270. All are true statements regarding use of sodium fluoride in the treatment of otosclerosis except? 
A. It inhibits osteblastic activity answer 
B. Used in active phase of otosclerosis when schwartz sign positive 
C. Has proteolytic activity(bone enzymes) 
D. 

271. A 5 year old child has burns on the surface of his body corresponding to the size of his palm. The percentage of
burns is? 
A. 1% answer 
B. 5% 
C. 10% 
D. 

272. False about pneumococcus is? 


A. Capsule aids in virulence answer 
B. Commonest cause of otitis media and pneumonia 
C. 
D. 

273. Medical treatment for variceal bleed is by? 


A. Octreotide 
B. Pantaprazole 
C. desmopressin??? 
D. 

274. Dental numbering is done by all except? 


A. FDI two digit system 
B. Anatomic and diagramatic charting 
C. Palmer notation 
D. 

275. Most important prognostic factor in congenital diaphragmatic hernia? 


A. Pulmonary hypertension……………answer 
B. Timing of surgery 
C. Size of defect 
D. 

276. Aflatoxin is produced by? 


A. Aspergillus flavus ……………..answer 
B. Aspergillus niger 
C. Candida 
D. 

277. A 7 month old child has bouts of cough ending with a whoop. What is the best way to confirm the diagnosis? 
A. Nasophayngeal swab …………………answer?? 
B. Cough sputum culture 
C. Tracheal aspirate 
D. 

279. Tetracycline is used in prophylaxis of? 


A. Cholera …………..answer 
B. Brucellosis 
C. Leptospirosis 
D. 

280. Orthopnoea in right heart failure develops due to? 


A. Reservoir function of pulmonary veins answer 
B. Pooling of blood in legs 
C. 
D. 

281. A young male presented with history of fever and a nodule in the leg. Histopathology of the nodule revealed foamy
histiocytes and neutrophillic infiltrate in the dermis. Most probable diagnosis is? 
A. Sweet's syndrome…………..answer 
B. Rosai Dorfman disease 
C. Erythema Nodosum Leprosum 
D. 

282. True regarding drug resistance of MRSA? 


A. Penicillinase enzyme production 
B. Due to change in penicillin binding receptors 
C. chrosome mediated answer 
D. Treated with amoxicillin + clavulanic acid 

283. Commonest cause for bilateral proptosis in children? 


A. Cavernous haemangioma 
B. Rhabdomyosarcoma………….answer? 
C. ALL 
D. AML 

284. Blood examination of a patient revealed low calcium, high phosphate and raised PTH. Which of the following
investigations need not be done? 
A. Urine microscopy answer 
B. PTH reassessment 
C. Vitamin D levels 
D. 

285. A 32 year old mountaineer has a hematocrit of 70%. What is the possible explanation? 
A. Polycythemia with dehydration…………..answer 
B. High altitude cerebral oedema 
C. High altitude pulmonary oedema 
D. Hemodilution 
286. False about C.diphtheriae is? 
A. Toxin production is chromosome mediated……………answer 
B. org cnfd by toxin production 
C. Toxic to heart and neurons 
D. 

287. True about gastric carcinoma is? 


A. Occult bleeding in stool is not seen 
B. associated with achlorhydria/hypochlorhydria………………….answer 
C. Always squamous cell carcinoma 
D. Radiosensitive 

288. Transwerfer of an amino group from an amino acid to an alpha keto acid is done by? 
A. Transweraminases ………answer 
B. Aminases 
C. Transwerketoses 
D. 

289. A patient presented with sudden onset of floaters and sensation of falling of a curtain in front of the eye. Which one
of the following is the appropriate diagnosis? 
A. Retinal detachment answer 
B. Eales disease 
C. Vitreous haemorrhage 
D. 

290. A girl presented with occipital headache associated with ataxia and vertigo. Mother also has similar complaints. Most
probable diagnosis is? 
A. Vestibular neuronitis 
B. Basillar migraine …………..answer 
C. 
D. 

291. Basal metabolic rate is closely associated with? 


A. Lean body mass ………….answer 
B. Body surface area 
C. Body mass index 
D. 

292. Arthropod transwermitted disease not found in India? 


A. West nile fever 
B. Dengue 
C. Yellow fever ………..answer 
D. 

293. Earliest symptom of GERD in an infant is? 


A. Respiratory distress answer 
B. Upper GI bleed 
C. regurgitation 
D.obstruction 

294. Cicatrising alopecia with perifollicular greying is most commonly associated with? 
A. Nail dystrophy 
B. Whitish lesion in the buccal mucosa 
C. Arthritis 
D. Discoid Plaques in the face answer 

295. Regarding Clostridium tetani, all are true except? 


A. Spores are resistant to heat answer 
B. 3 doses give immunity in primary immunisation 
C. Incubation period is 6-10 days 
D. Person to person transwermission does not occur 

296. Drug of choice for central diabetes insipidus? 


A. Desmopressin ……………answer 
B. Leuprolide 
C. Thiazide 

297. in 7 yr old following Sx for craniopharyngioma hormone first to b given 


Growth hormone ???? 
Steroid 
Prolactin 
ACTH 
298. hypothyroidsum is seen in ... 
Hasimoto thyroidits ???? 

299 one on diagnosis of diabetis isipidus all except .i dont remember options 

300. most useful in acute illness 


a.case fatalityrate ans 

301 .one on mri contrast 


a.gadollinium crosses BBB 
b.test dose should be 
c. 
302. campylobacter jejuni a/e 
a.humans r reservoirs ans (repeat) 

303 .all are used in diabetic macular edema except 


a.tamoxifen ans 

304. common cause of bilateral proptposis 


a.chloroma ans??? 
there were two questions on b/l proptosis 

305. true about prion 


a.catalyses folding of other proteins ans 
totally there were three questions on prion 

306. free radicals generated by all except 


a.superoxide dismutase ans 

307. best bactericidal system 


a.cation basic protein 
b. oxygen metabolite ans 
there werwe 2 similar question oxidase is ans for the

You might also like